You are on page 1of 41

І

1. An infant was born with body mass 3 kg and body length 50 cm. Now he is 3 years old. His
brother is 7 years old, suffers from rheumatic fever. Mother asked a doctor for a cardiac
check up of the 3-year-old son. Where is the left relative heart border located?
A. 1 cm left from the left medioclavicular line
B. 1 cm right from the left medioclavicular line
C. Along the left medioclavicular line
D. 1 cm left from the left parasternal line
E. 1 cm right from the left parasternal line
Answer A: According to anatomical and physiological features in children aged 2-7 years the left
relative heart border is determined of 1 cm to the left medioclavicular line.
2. A baby was born at 36 weeks of gestation Delivery was normal, by natural way. The baby
has a large cephalohematoma. The results of blood count are: Hb- 120g/l, Er- 3, 5 · 1012/l,
total serum bilirubin - 123 mmol/l, direct bilirubin - 11 mmol/l, indirect - 112 mmol/l. What
are the causes of hyperbilirubinemia in this case?
A. Erythrocyte hemolysis
B. Intravascular hemolysis
C. Disturbance of the conjugative function of liver
D. Bile condensing
E. Mechanical obstruction of the bile outflow
Answer A: A large cephalohematoma in a newborn is the cause of hemolysis of red blood cells
and, as a consequence, hyperbilirubinemia.
3. An 18-year-old patient was admitted to a hospital with complaints of headache, weakness,
high fever, sore throat. Objectively: enlargement of all groups of lymph nodes was revealed. The
liver is enlarged by 3 cm, spleen - by 1 cm. In blood: leukocytosis, atypical lymphocytes - 15%.
What is the most probable diagnosis?
A. Infectious mononucleosis
B. Acute lymphoid leukosis
C. Diphtheria
D. Angina
E. Adenoviral infection
Answer A: Infectious mononucleosis is characterized by such symptoms as polyadenopathy,
high fever, sore throat, hepatosplenomegaly, leukocytosis and atypical lymphocytes
(mononuclears) in blood.
4. A 9-month-old child presents with fever, cough, dyspnea. The symptoms appeared 5 days ago
after a contact with a person having URTI. Objectively: the child is in grave condition.
Temperature of 38oC, cyanosis of nasolabial triangle is present. Respiration rate - 54/min, nasal
flaring while breathing. There was percussion dullness on the right below the scapula angle, and
tympanic sound over the rest of lungs. Auscultation revealed bilateral fine moist rales (crackles)
predominating on the right. What is the most likely diagnosis?
A. Acute pneumonia
B. URTI
C. Acute laryngotracheitis
D. Acute bronchitis
E. Acute bronchiolitis
Answer A: The most likely diagnosis is acute pneumonia because the child has such symptoms
as percussion dullness on the right below the scapula angle, fine moist rales (crackles)
predominating on the right, symptoms of respiratory insufficiency (cyanosis of nasolabial
triangle, respiration rate - 54/min, nasal flaring while breathing).
5. A 10-year-old boy complains of pain in his left eye and strong photophobia after he has
injured his left eye with a pencil at school. Left eye examination revealed: blepharospasm, ciliary

1
and conjunctival congestion, cornea is transparent, other parts of eyeball have no changes. Visus
0,9. Right eye is healthy, Visus 1,0. What additional method would you choose first of all?
A. Staining test with 1% fluorescein
B. X-ray examination of orbit
C. Tonometria
D. Gonioscopia
E. Cornea sensation-test
Answer A: The staining test with 1% fluorescein allows to diagnose corneal ulcers and erosion.
6. A 13-year-old girl complains of periodic prickly pain in the heart region. Percussion revealed
no changes of cardiac borders. Auscultation revealed arrhythmic enhanced heart sounds,
extrasystole at the 20-25 cardiac impulse. ECG showed the sinus rhythm, impaired
repolarization, single supraventricular extrasystoles at rest. What is the most likely diagnosis?
A. Somatoform autonomic dysfunction
B. Rheumatism
C. Nonrheumatic carditis
D. Myocardial degeneration
E. Intoxication syndrome
Answer A: Somatoform autonomous dysfunction is characterized by absent fever, any
inflammatory changes in blood and urine tests against periodic prickly pain in the heart, sinus
rhythm, single supraventricular extrasystoles and impaired repolarization.
7. An 8-year-old boy during preventive examination was determined to have changes in his spine
curvature in frontal plane: the right shoulder is lowered and flat, scapulae angles are of different
height due to the right scapula being shifted down. Waist triangles are pronounced on the both
sides; longitudinal muscles of the back form muscle cushion on the left. What type of posture is
detected in the child?
A. Scoliotic
B. Kyphotic
C. Lordotic
D. Stooping
E. Corrected
Answer A: The described changes in the frontal plane are characteristic for scoliosis.
8. A 9-year-old boy has been suffering from bronchoectasis since he was 3. Exacerbations
occur quite often, 3-4 times a year. Conservative therapy results in short periods of remission.
The disease is progressing, the child has physical retardation. The child’s skin is
pale, acrocyanotic, he has "watch glass"nail deformation. Bronchography revealed saccular
bronchiectases of the lower lobe of his right lung. What is the further treatment tactics?
A. Surgical treatment
B. Further conservative therapy
C. Physiotherapeutic treatment
D. Sanatorium-and-spa treatment
E. Tempering of the child’s organism
Answer A: Considering the progressing of the disease, the child's physical retardation, the
symptoms of systemic hypoxia (the child's skin is pale, acrocyanotic, he has "watch glass" nail
deformation), as a further tactic it is advisable to choose surgical treatment.
9. A 7-year-old child complains of cramping pain occuring after mental exertion, cold drinks
and eating ice-cream. Instrumental examination allowed to diagnose biliary dyskinesia of
hypertensive type. What group of drugs should be prescribed for treatment?
A. Antispasmodics and choleretics
B. Choleretics and cholekinetics
C. Sedatives and cholekinetics
D. Antioxydants
E. Antibiotics
2
Answer A: According to the protocols for the treatment of dyskinesia bile ducts in children with
hypertonic type of dyskinesia it is advisable to prescribe antispasmodics and choleretics.

10. 2 weeks after recovering from tonsillitis an 8-year-old boy developed oedemas of face and
lower limbs. Objectively: the patient is in grave condition, BP - 120/80 mm Hg. Urine is of dark
brown colour. Oliguria is present. On urine analysis: specific gravity - 1,015, protein - 1,2
g/l, RBCs are leached and cover the whole vision field, granular casts - 1-2 in the vision field,
salts are represented by urates (large quantity). What is the most likely diagnosis?
A. Acute glomerulonephritis with nephritic syndrome
B. Acute glomerulonephritis with nephrotic syndrome
C. Acute glomerulonephritis with nephrotic syndrome, hematuria and hypertension
D. Acute glomerulonephritis with isolated urinary syndrome
E. Nephrolithiasis
Answer A: BP elevation + odemas (both are extrarenal clinical symptoms) + urinary syndrome =
nephritic syndrome. nephrotic syndrome – without arterial hypertension, must be presence of
gross proteinuria.
11. A 14-year-old girl has been presenting with irritability and tearfulness for about a year. A
year ago she was also found to have diffuse enlargement of the thyroid gland (II grade). This
condition was diagnosed as a pubertal manifestation, the girl did not undergo any treatment. The
girl’s irritability gradually gave place to a complete apathy. The girl got puffy face, soft tissues
pastosity, bradycardia, constipations. Skin pallor and gland density progressed, the skin got a
waxen hue. What disease may be assumed?
A. Autoimmune thyroiditis
B. Diffuse toxic goiter
C. Thyroid carcinoma
D. Subacute thyroiditis
E. Juvenile basophilism
Answer A: stages of autoimmune thyroiditis: hyperfunction was present 1 year ago. Progression
of the disease to hypofunction stage.
12. A 12-year-old cild has been hit on the stomach. The patient is in moderately grave condition,
has a forced position in bed. The skin is pale, heart rate - 122/min. Stress on the left costal arch
causes a slight pain. Wei- nert and Kulenkampff symptoms are positive. Macroscopically the
urine is unchanged. What is the most likely diagnosis?
A. Spleen rupture, abdominal bleeding
B. Left kidney rupture, retroperitoneal hematoma
C. Rupture of the pancreas
D. Liver rupture, abdominal bleeding
E. Rupture of a hollow organ, peritonitis
Answer A: localization of the pain, absence of hematuria, anamnesis like closed abdominal
trauma
13. A 4-year-old boy was vaccinated in violation of his vaccination schedule. There are
complaints of pain in the throat during swallowing, headache, inertness, fever. Objecti-
vely: the child is pale; anterior cervical lymphnodes are enlarged; tonsils are swollen, with
cyanotic hyperemy, and covered with grey-white coating that cannot be removed; if it is
forcibly removed, tonsils bleed. The most likely diagnosis is:
A. Oropharyngeal diphtheria
B. Lacunar tonsillitis
C. Pseudomembranous (Vincent’s) tonsillitis
D. Infectious mononucleosis
E. Follicular tonsillitis
Answer A: plaques cannot be removed, intoxication, lymph nodes enlargement.

3
14. A child is 3-week-old. Since his birth periodical vomiting is observed occuring several
minutes after feeding. Vomit mass does not exceed previous feeding volume. Body mass is
appropriate to the child’s age. What is the most likely cause for the symptoms described?
A. Pylorospasm
B. Esophageal chalasia
C. Adrenogenital syndrome
D. Pylorostenosis
E. Esophageal achalasia
Answer A: absence of malnutrition, symptoms from other systems of organs,
15. An 8-year-old girl periodically has sudden short-term heart pain, sensation of chest
compression, epigastric pain, dizziness, vomiting. Objectively: the patient is pale, respiratory
rate - 40/min, jugular pulse is present. Heart rate - 185 bpm, of poor volume. BP - 75/40 mm Hg.
ECG taken during an attack shows ectopic P waves, QRS wave is not deformed. At the end of an
attack a compensatory pause is observed. The most likely cause of the attack is:
A. Paroxysmal atrial tachycardia
B. Sinus tachycardia
C. Paroxysmal ventricular tachycardia
D. Complete AV-block
E. Atrial fibrillation
Answer A: accelerated heart rate, ectopic P waves, QRS wave is not deformed.
16. During the newborn infant’s examination redness of the skin was detected, which occurred
immediately after birth and became the most pronounced on the second day of the infant’s life.
What provisional diagnosis can be made?
A. Simple erythema
B. Toxic erythema
C. Transient erythema
D. Erythema nodosum
E. Erythema anulare centrifugum
Answer A: presence at once after birth, increased till several days of life.
17. An 8-year-old boy developed a temperature of 37, 5oC two days after his recovery from
the case of URTI. He complains of suffocation, heart pain. Objectively: the skin is pale,
tachycardia, the I heart sound is weakened, short systolyc murmur in the 4th intercostal
area near the left edge of the breastbone. What heart disorder such clincal presentation is
characteristic of?
A. Nonrheumatic myocarditis
B. Primary rheumatic carditis
C. Myocardiodystrophy
D. Fallot’s tetrad
E. Cardiomyopathy
Answer A: connection with URTI, systolic murmur, cardiomegaly.
18. An infant has been born at the 41st week of gestation. The pregnancy was complicated
with severe gestosis of the second semester. The weight of the baby is 2400 g, the height is
50 cm. Objectively: the skin is flabby, the layer of subcutaneous fat is thin, hypomyotonia,
neonatal reflexes are weak. The internal organs are without pathologic changes. This
newborn can be estimated as a:
A. Full-term infant with prenatal growth retardation
B. Premature infant
C. Immature infant
D. Postmature infant
E. Full-term infant with normal body weight
Answer A:: term 38-42 is normal, weight / height 48 (normal is 60).

4
19. Mother of an 8-year-old girl complains that the child is too short and has excessive body
weight. Objectively: obesity with fat deposits on the torso and face (round moon-like face), acne,
striae on the thighs and lower abdomen, hirsutism.What hormone can cause such symptoms,
when in excess?
A. Cortisol
B. Thyroxin
C. Testosterone
D. Insulin
E. Glucagon
Answer A: round moon-like face is specific symptom of the Cushing syndrome occurs due to
abnormally high levels of the hormone cortisol. Other additional symptoms were presented such
as obesity with fat deposits on the torso and face, acne, striae on the thighs and lower abdomen,
hirsutism. Insulin and glucagon are excluded as there is no information regarding serum glucose
level or problems with polyuria or polydipsia. Thyroxin is excluded also as hirsutism and striae
are not specific for it. Testosterone is not the correct answer as there is no information about
ambiguous genitalia.
20. A 9-year-old girl complains of fever up to 37,5oC, headache, inertness, weakness, loss
of appetite, stomachache, and frequent painful urination. Provisional diagnosis of acute
pyelonephritis ismade.Clinical urine analysis: specific gravity - 1018, no protein, leukocytes
- 10-15 in the vision field. What investigation method can verify the diagnosis of urinary
system infection?
A. Bacteriological inoculation of urine
B. Rehberg test (creatinine clearance test)
C. Zymnytsky test (measurement of daily
diuresis)
D. Complete blood count
E. Clinical urine analyses, dynamic testing
Answer A: due to the fact that leucocytes quantity in the urine is slightly higher than in norm
(10-15 per field of view), UTI can be also verified with bacteriologic tests. creatinine clearance
test, Zymnytsky test, Complete blood count do not give us any information regarding
inflammation in the UTI.
21. On the 3rd day of life a newborn, who had suffered birth asphyxia, developed hemorrhage
from the umbilical wound. Laboratory analysis reveals hypocoagulation, thrombocytopenia, and
hypothrombinemia. What is the cause of such clinical developments?
A. Disseminated intravascular coagulation
B. Hemorrhagic disease of newborn
C. Congenital angiopathy
D. Thrombocytopenic purpura
E. Umbilical vessel trauma
Answer A: The patient has no skin eruptions so this is not Thrombocytopenic purpura. There is
no information regarding anemia that is why Hemorrhagic disease of newborn is excluded.
Congenital angiopathy and trauma affect only vessels, but not coagulation system or
thrombocytes. The child had asphyxia and this was factor for Disseminated intravascular
coagulation start with systemic changes in the coagulation system.
22. A 14-year-old girl has been presenting with irritability and tearfulness for about a year.
A year ago she was also found to have diffuse enlargement of the thyroid gland (II grade).
This condition was regarded as a pubertal manifestation, the girl did not undergo any
treatment. The girl’s irritability gradually gave place to a complete apathy. The girl developed
puffy face, soft tissues pastosity, bradycardia, constipations. Skin pallor and gland density
progressed, the skin developed a waxen hue. What disease can be suspected?
A. Autoimmune thyroiditis
B. Diffuse toxic goiter
5
C. Thyroid carcinoma
D. Subacute thyroiditis
E. Juvenile basophilism
Answer A: As it was given in the task that the 14-year-old girl had diffuse enlargement of the
thyroid gland (II grade) and regarded as a pubertal manifestation, complete apathy, developed
puffy face, soft tissues pastosity, bradycardia, constipations, skin pallor and gland density
progressed. This is clinical picture autoimmune thyroiditis.
23. A 2-year-old girl has a medical history of recurrent obstructive pneumonia. In the
lungs various moist and dry crackles can be auscultated, breath sounds are dimini-
shed. Sputum is thick, viscous and difficult to expectorate. Drumstick fingers and physi-
cal developmental retardation are observed. What preliminary diagnosis can be made?
A. Pulmonary mucoviscidosis
B. Recurrent bronchitis
C. Bronchial asthma
D. Congenital polycystic lungs
E. Pulmonary tuberculosis
Answer A: As it was given in the task that the child had recurrent obstructive pneumonia and dry
crackles can be auscultated, breath sounds are diminished, sputum is thick, viscous and difficult
to expectorate. The child had drumstick fingers and physical developmental retardation are
observed. This is congenital lung disease can be suspected.
24. A 9-month-old child presents with fever, cough, dyspnea. The symptoms appeared
5 days ago after a contact with a person suffering from URTI. Objectively: the child
is in grave condition. Temperature of 38oC, cyanosis of nasolabial triangle is present.
Respiration rate - 54/min., nasal flaring during breathing. There was percussion dullness
on the right below the scapula angle, and tympanic sound over the rest of lungs. Auscultation
revealed bilateral fine moist crackles predominating on the right. What is the most likely
diagnosis?
A. Acute pneumonia
B. URTI
C. Acute laryngotracheitis
D. Acute bronchitis
E. Acute bronchiolitis
Answer A: As it was given in the task that the child had fever, cough, dyspnea, percussion
dullness on the right below the scapula angle and аuscultation revealed bilateral fine moist
crackles predominating on the right. This is clinical picture аcute pneumonia
25. After a case of purulent otitis a 1-year-old boy has developed pains in the upper third of
the left thigh, body temperature up to 39oC. Objectively: swelling of the thigh in its upper
third and smoothed out inguinal fold. The limb is in semiflexed position. Active and passi-
ve movements are impossible due to severe pain.What diagnosis is most likely?
A. Acute hematogenous osteomyelitis
B. Acute coxitis
C. Intermuscular phlegmon
D. Osteosarcoma
E. Brodie’s abscess
Answer A: osteomyelitis is an infection of the bones that occurs either via hematogenous spread
(most common in children). Clinical Presentation - most common findings in children with
osteomyelitis are pain of affected area and loss of function, however 2 distinct clinical
syndromes have been described. Children presenting with fever, localized pain, who appear
acutely ill (likely septic). A more indolent course, with gradual onset of pain and concurrent loss
of function. In this presentation, the child may be afebrile or have low-grade fevers. As the lower
extremity is more commonly affected, a common presentation is a child with a limp
26. A child is 1 month 10 days old. ”Gushing” vomiting has been observed since the age of 3
6
weeks. The vomit volume exceeds the volume of the previous feeding. Objectively: the chi-
ld is inert. Skin elasticity and tissue turgor are decreased. Hour-glass deformity sign is
positive. The preliminary diagnosis is pyloric stenosis. What treatment tactics should be
chosen?
A. Surgery
B. Prescription of Cerucal (Metoclopramide)
C. Atropinization
D. Internal administration of Novocaine
E. Prescription of Pipolphen
Answer A: Pyloric stenosis is an uncommon condition in infants that blocks food from entering
the small intestine. The causes of pyloric stenosis are unknown, but genetic and environmental
factors might play a role. Signs and symptoms include: vomiting after feeding; рersistent hunger;
stomach contractions; dehydration;  changes in bowel movements;  weight problems. Surgery
cures pyloric stenosis.
27. A 16-year-old adolescent has been hospitalized with complaints of unceasing nasal
hemorrhage and unbearable pain in his right elbow joint. Objectively: the large joint
is enlarged and defigurated, the skin over the joint is hyperemic. Arthropathy signs can
be observed in the other joints. Ps- 90/min. Blood test: erythrocytes - 3, 9 · 1012/l, Нb- 130
g/l, color index - 1,0, leukocytes - 5, 6 · 109/l, platelets - 220 · 109/l, ESR- 6 mm/hour.
Lee-White coagulation time: start- 24 , end-2710. What drug would be most efficient in
the treatment of this patient?
A. Cryoprecipitate
B. Calcium chloride
C. Concentrated red cells
D. Aminocapronic acid
E. Vicasol (Menadione)
Answer A: cryoprecipitate, also called cryo for short, is a frozen blood product prepared from
blood plasma. One of the most important constituents is factor VIII (also called antihaemophilic
factor or AHF), which is why cryoprecipitate is sometimes called, or refined into,
cryoprecipitated antihaemophilic factor or cryoprecipitated AHF. Indications for giving
cryoprecipitate include haemophilia – used for emergency back up when factor concentrates are
not available.
28. An 8-year-old girl periodically has sudden short-term heart pains, sensation of chest
compression, epigastric pain, dizziness, vomiting. Objectively: the patient is pale, respi-
ratory rate - 40/min., jugular pulse is present. Ps- 185/min., of poor volume. BP- 75/40
mm Hg. ECG taken during an attack shows ectopic P waves, narrow regular QRS waves.
At the end of an attack a compensatory pause is observed. The most likely cause of the
attack is:
A. Paroxysmal atrial tachycardia
B. Sinus tachycardia
C. Paroxysmal ventricular tachycardia
D. Complete AV-block
E. Atrial fibrillation
Answer A: It is episode of paroxysmal tachycardia because it starts spontaneously and finished
spontaneously turning into normal rhythm; during attack of PT condition can be changed
producing unusual sensation of palpitation in chest, dizziness and even syncope, it can produce
dyspeptic sighns; HR is more than 150 \min that is very typical for the PT attack. It originate
from atrium because we get presentation of P wave with changed morphology concerning sinus
origin that is typical for atrium pacemakers.
29. A 10-year-old patient has a history of mild bronchial asthma. During a regular check-up
the patient should be recommended:
A. To avoid allergenic food
7
B. To avoid body tempering procedures
C. To avoid sports
D. To avoid spa treatment
E. To avoid going to the seaside
Answer A: If mild grade of BA was diagnosed the first recommendation will concern to
elimination therapy i.e. to avoid contact or consuming reasonable allergens that produced
sensibilization in patient
30. A baby was born by a young smoker. The labour was complicated by uterine inertia,
difficult delivery of the baby’s head and shoulders. The baby’s Apgar score was 4.
Which of the following is a risk factor for a spinal cord injury?
A. Difficult delivery of the head and shoulders
B. Young age of the mother
C. Pernicious habits
D. Uterine inertia
E. Chronic hypoxia
Answer A: Prolonged delivering of head and shoulders is the most significant factor because
this mechanism responsible for mechanical, hypoxic trauma altogether.

31.An 8-year-old boy developed a temperature of 37, 5oC two days after his recovery from
the case of URTI. He complains of suffocation, heart pain. Objectively: the skin is pale,
tachycardia, the I heart sound is weakened, short systolyc murmur in the 4th intercostal
area near the left edge of the breastbone. What heart disorder such clincal presentation is
characteristic of?
A. Nonrheumatic myocarditis
B. Primary rheumatic carditis
C. Myocardiodystrophy
D. Fallot’s tetrad
E. Cardiomyopathy
Answer A: There are clinical nonspecific signs of cardiac damage: paleness, suffocation, heart
pain, tachycardia, appearance of systolic murmur. Taking into account anamnestic data of URTI
preceeding to this symptoms the most probable diagnosis is viral carditis.
32. A 9-year-old boy has been suffering from multiple bronchiectasis since he was 3
years old. Exacerbations occur frequently (3-4 times a year), after conservative therapy
there are short remission periods. The disease progresses, the child is physically
underdeveloped, presents with pale skin, acrocyanosis, deformed nail plates in the
shape of ”clock-face”. Bronchography reveals saccular bronchiectases in the lower lobe of
the right lung. What further treatment tactics should be chosen?
A. Surgical intervention
B. Continuation of conservative therap
C. Physiotherapy
D. Sanatorium-and-spa treatment
E. Physical training
Answer A: Clinical findings confirm presence of local chronic pneumonia accompanied by
bronchial rebuilding that predispose for recurrent re-exacerbation of lungs and bronchi. The only
effective way to solve it is surgical removing of affected segment or lobe with affected bronchi.
33. An infant has been born at the 41st week of gestation. The pregnancy was complicated
with severe gestosis of the second semester. The weight of the baby is 2400 g, the height is
50 cm. Objectively: the skin is flabby, the layer of subcutaneous fat is thin, hypomyotonia
is observed, neonatal reflexes are weak. The internal organs are without pathologic
alterations. This newborn can be assessed as a:
A. Full-term infant with prenatal growth retardation
B. Premature infant
8
C. Immature infant
D. Postmature infant
E. Full-term infant with normal body weight
Answer A: According to the term of gestation child was full term (38-41 week is normal
gestational term), but clinical signs indicate retardation of child maturation: low weight,
undeveloped reflexes, low muscular tone, flabbiness.
34. After a case of purulent otitis a 1-year-old boy has developed pain in the upper third of
the left thigh, body temperature up to 39oC. Objectively: swelling of the thigh in its upper
third and smoothed out inguinal fold. The limb is in semiflexed position. Active and passive
movements are impossible due to severe pain. What diagnosis is the most likely?
A. Acute hematogenous osteomyelitis
B. Acute coxitis
C. Intermuscular phlegmon
D. Osteosarcoma
E. Brodie’s abscess
Answer A: Ta king into account preexisting purulent inflammatory process, involved bone
(limb) into pathologic process, signs of inflammation, severe pain, limitation in movements the
most, absence of soft tissue purulent affection only reactive edema of tissues the most probable
diagnosis is acute hematogenous osteomyelitis
35. A 10-year-old boy with symptoms of arthritis and myocarditis was delivered into
a hospital. Based on clinical examination the preliminary diagnosis of juvenile rheumatoid
arthritis was made. What symptoms are the most contributive for the diagnostics of this di-
sease?
A. Morning stiffness in affected joints
B. Regional hyperemia of the joints
C. Affection of the large joints
D. Enlarged heart
E. Increased heart rate
Answer A: For the clinical feature of JRA (juvenile rheumatoid arthritis) the most contributive
will be the symptom of morning stiffness.
36. A 9-month-old child presents with fever, cough, dyspnea. The symptoms appeared 5
days ago after a contact with a person with URTI. Objectively: the child is in grave
condition. Temperature is 38 C, cyanosis of nasolabial triangle is present. RR- 54/min,
nasal flaring during breathing is observed. There was percussion dullness on the right
below the scapula angle and tympanic sound over the other areas of lungs. Auscultati-
on revealed bilateral fine moist crackles predominating on the right. What is the most
likely diagnosis?
A. Pneumonia
B. URTI
C. Acute laryngotracheitis
D. Acute bronchitis
E. Acute bronchiolitis
Answer A: Taking into account presence of clinical confirmative symptoms for pneumonia like
topical percussion dullness, topical crepitation or moist rales, signs of respiratory insufficiency,
severe intoxication presentation after URTI the most probable diagnosis is pneumonia. It must be
confirmed radiologically.
37. The mother of a 3-month-old child came to a family doctor with complaints of her
child being physically underdeveloped and suffering from cough attacks and dyspnea.
Anamnesis: the child is the result of the second full-term pregnancy with the risk of
miscarriage (the first child died of pulmonary pathology at the age of 4 months, according
to the mother). Body mass at birth is 2500 g. Cough attacks were observed from the first days of
life, twice the child was treated for bronchitis. Considering the severity of the child’s condition
9
the doctor made the referral for hospitalization. What diagnosis was most likely stated in the
referral?
A. Mucoviscidosis (Cystic fibrosis)
B. Acute obstructive bronchitis
C. Recurrent obstructive bronchitis
D. Pertussis
E. Acute obstructive pneumonia
Answer A: As it was given in the task that the child had persistent problems with the respiratory
system in this regard acute pneumonia and acute bronchitis are diagnosis of exclusion. It is not
specific for Recurrent obstructive bronchitis and pertussis to cause underdevelopment or
undernutrition, but this can be seen in patients with Cystic fibrosis. Even more, it was given, that
mother had other baby with pulmonary pathology, hence some congenital lung disease can be
suspected.
38. A boy was born at 32 weeks of gestation. 2 hours after the birth he developed respiratory
distress (RD). The RD severity assessed by Silverman score was 5. The respiratory disorders
progressed, respiratory failure could not be eliminated by Martin-Bouyer CPAP (continuous
positive airway pressure). X-ray of lungs shows reticular and nodular pattern, air bronchogram.
What is the most likely cause of respiratory distress syndrome?
A. Hyaline membrane disease
B. Segmental atelectasis
C. Bronchopulmonary dysplasia
D. Congenital pulmonary emphysema
E. Edematous hemorrhagic syndrome
Answer A: the commonest cause of the RD in few hours after the birth is Hyaline membrane
disease, which was proved by specific reticular and nodular pattern, air bronchogram on the X-
ray of lungs. In this task it was indicated that the child was born prematurely, hence RD probably
is caused by surfactant deficiency. This was proved by progressed respiratory failure, that could
not be eliminated with Martin-Bouyer CPAP as in such cases they have collapse of the terminal
air-spaces throughout the normal cycle of inhalation and exhalation. Segmental atelectasis is
incorrect answer as on the X-ray diffuse changes were described. Bronchopulmonary dysplasia is
incorrect answer as it can be considered as the consequence of the RD, but in this task the
emergency condition developed during first day of life. Congenital pulmonary emphysema and
Edematous hemorrhagic syndrome are incorrect answers as the symptoms appeared in 2 hours
after the birth, but they were absent at the birth.
39. An 8-year-old child with a 3-year-long history of diabetes was hospitalized in hyperglycemic
coma. Specify the initial dose of insulin to be administered:
A. 0,1-0,2 U/kg of body weight per hour
B. 0,05 U/kg of body weight per hour
C. 0,2-0,3 U/kg of body weight per hour
D. 0,3-0,4 U/kg of body weight per hour
E. 0,4-0,5 U/kg of body weight per hour
Answer A: Current guidelines recommend administering insulin 0,1 U/kg of body weight per
hour. In the early 1970s, numerous studies demonstrated that “low-dose” or “physiological” (0.1
U/ kg/ h) doses of insulin were effective in controlling Diabetic ketoacidosis. Between 1976 and
1980, however, numerous prospective randomized studies in adults and children demonstrated
the efficacy of lower or physiological doses of insulin by various routes of therapy, which, unlike
the high-dose protocol, were associated with a lower incidence of hypokalemia and
hypoglycemia. The average glucose decrement under such low-dose protocols was between 75
and 120 mg · dl-1 · h-1, which was very similar to the response to larger doses of insulin. Because
of the similar metabolic response to high or low doses of insulin, it was questioned whether
Diabetic ketoacidosis patients were significantly more insulin resistant than well-controlled type

10
1 diabetic patients. Dose 0,05 U/kg of body weight per hour is not initial and can be used after
decrease of the glucose level to 13-14 mmol/L.
40. A 17-year-old young man complains of general weakness, trismus, twitching of the
muscles in his left shin. 7 days ago he pierced his foot with a nail. Objectively: at the sole
of the foot there is a wound, 0,3х0,2 mm in size, with small amount of serous-purulent
discharge, the skin around the wound is hyperemic.What is the most likely diagnosis?
A. Tetanus
B. Phlegmon
C. Osteomyelitis
D. Infected wound
E. Erysipelas
Answer A: Despite the described local symptoms, Phlegmon, infected wound and Osteomyelitis
are not correct answers as the patient has general weakness and trismus, which is not specific for
these pathology. But trismus and twitching of the muscles are the symptoms of the Tetanus
manifestation. Furthermore incubation period in 7 days and puncture wound (0,3х0,2 mm in
size) are additional indicators of the Tetanus. The skin around the wound was hyperemic and the
patient had weakness but there was no information about well-demarcated skin rash thus
Erysipelas can be excluded.
41. An infant is 2,5 months old. The onset of the disease was gradual, the child had normal
body temperature but presented with slight cough. Within a week the cough intensified,
especially at night; on the 12th day the child developed cough fits occurring up to 20 times
per day and followed by vomiting. There was one instance of respiratory arrest. Make the
diagnosis:
A. Pertussis
B. Parainfluenza
C. Congenital stridor
D. Respiratory syncytial infection
E. Adenovirus infection
Answer A: As it was give in the task that the symptoms were mild at the start and gradually
worsened, thus Parainfluenza, Respiratory syncytial infection and Adenovirus infection can be
excluded as they usually manifest with acute symptoms which gradually become less. Congenital
stridor symptoms are usually seen just after the birth as there is constant change of the structure
and as usual they do not have any differences between day and night time. On the other hand,
Pertussis in infants usually has shorter phases and, as it is in this patient, initial period with mild
symptoms can last only a week. Night attacks and respiratory arrest are also very typical for
Pertussis.
42. An infant is 3 weeks old. Since birth there has been observed periodical vomiting within a
few minutes after feeding. The amount of vomitive masses does not exceed the volume of
previous feeding. The infant has age-appropriate body weight. What is the most likely cause of
this symptom?
A. Pylorospasm
B. Esophageal chalasia
C. Adrenogenital syndrome
D. Pyloristenosis
E. Esophageal achalasia

Answer A: It is typical for patients with Chalasia and Pylorostenosis to have massive vomit, but
in this task the patient did not have persistent massive vomit, it was periodical and did not
exceed the volume of previous feeding. Hence these two answers can be excluded. Vomiting
started within a few minutes after feeding but not during the feeding, that is why achalasia is
incorrect answer as well. There is no information about ambiguous genitalia or dehydration or

11
17α-hydroxyprogesterone blood levels to diagnose Adrenogenital syndrome. And this disease is
not the most likely cause as it prevalence is not high.
43. A newborn with gestational age of 31 weeks presents with hypotonia and depressed
consciousness. Hematocrit is 35%, general cerebrospinal fluid analysis shows increased
content of erythrocytes and protein, and low glucose. These data correspond with the clinical
presentation of:
A. Intracranial hemorrhage
B. Meningitis
C. Sepsis
D. Anemia
E. Prenatal infection
Answer A: gestation age less than 35 weeks is risk factor of the intracranial hemorrhage, which
is proved by the indicated symptoms (hypotonia and depressed consciousness) and results of the
cerebrospinal fluid analysis (erythrocytes and protein). No information about leucocytes in the
fluid was given, hence meningitis, infection and sepsis can be excluded. Patients with anemia
won’t have any changes in the cerebrospinal fluid that is why anemia can be excluded as well.
44. From urine of a 14-year-old boy with the exacerbation of secondary obstructive
pyelonephritis Pseudomonas aeruginosa was isolated with a titer of 1000000microbes per
1 ml. What antibiotic is the most advisable in this case?
A. Ciprofloxacin
B. Ampicillin
C. Cefazolinum
D. Azithromycin
E. Chloramphenicol
Answer A: Pseudomonas aeruginosa is not the commonest etiology of the pyelonephritis and it is
as usual resistant to the antibacterial drugs of the first line. According to the guidelines urinary
tract infection associated with Pseudomonas aeruginosa can be treated with fluoroquinolones.
Age less than 14 y.o. is prohibition to the administration of such medical drugs, but in this task
the boy is 14 y.o. that is why Ciprofloxacin is the correct answer.
45. A 3-year-old girl is being treated at a resuscitation unit with diagnosis ”acute kidney failure,
oligoanuric stage” . ECG: high T wave, extended QRS complex, displacement of S-T interval
downwards below the isoline. What electrolyte imbalance is it?
A. Hyperkalemia
B. Hypokalemia
C. Hypocalcemia
D. Hypercalcemia
E. Hyperphosphatemia
Answer A: in the case of the acute renal failure the most possible condition from the listed above
is Hyperkalemia, which was proved by ECG high T wave, extended QRS complex, displacement
of S-T interval downwards below the isoline.

ІІ
1. A 2-year-old child in a satisfactory condition periodically presents with moderate
proteinuria, microhematuria. US results: the left kidney is undetectable, the right one is
enlarged, there are signs of double pyelocaliceal system What study is required to specify
the diagnosis?
A. Excretory urography
B. Micturating cystography
C. Retrograde urography
D. Doppler study of renal vessels
E. Radioisotope renal scan

12
Correct anser is A - Excretory urography.
Explanation: Urography is a radiologic technique used for the evaluation of the genitourinary
system—specifically, the kidneys, ureters, and bladder. Although originally performed using
plain radiographic techniques, advanced imaging modalities have been progressively refined
such that computed tomography (CT) and/or magnetic resonance imaging (MRI) have largely
replaced excretory urography (EU) as the optimal way to image the genitourinary system. 
Indications:
Excretory urography
The following are accepted indications for EU, as delineated by the American College of
Radiology (ACR): [7]
 To evaluate the presence or continuing presence of suspected or known ureteral
obstruction.
 To assess the integrity of the urinary tract status post trauma (including iatrogenic
interventions), particularly in situations in which cross-sectional imaging is unavailable or
inappropriate.
 To assess the urinary tract for suspected congenital anomalies, particularly in situations in
which cross-sectional imaging is unavailable or inappropriate.
 To assess the urinary tract for lesions that may explain hematuria or infection. In particular,
EU may be used to evaluate for an underlying parenchymal mass or may be used to
evaluate for a lesion of the urothelial tract in settings in which cross-sectional imaging is
unavailable or inappropriate
Contraindications
Excretory urography
The technique used in performing a complete EU examination is discussed in detail later;
however, the basic premise involves obtaining plain radiographic images of the abdomen at
various time points after the administration of intravenous iodinated contrast material. In
addition, a compression device is often used for more optimal visualization of the collecting
system. Thus, contraindications to EU pertain to the use of iodinated contrast material as well as
the use of the compression paddle. A positive pregnancy test is an additional absolute
contraindication to this procedure.
Risk factors pertaining to use of iodinated contrast material are as follows [25] :
 Allergy: A history of a prior allergic-like reaction to contrast media is associated with an up
to 5-fold increase in the likelihood of experiencing a subsequent reaction. [26] In addition,
any patient with a predilection to allergic reactions may predispose them to a reaction after
the administration of contrast media. Given the increased risk of severe life-threatening
anaphylaxis related to the administration of contrast media in the setting of history of
atopy, the risk versus benefits should be discussed before following through with the
procedure. A premedication regimen may be used to reduce the risk of anaphylaxis.
 Asthma: A history of asthma may be indicative of a higher likelihood of developing a
contrast reaction. [
 Cardiac status: Attention must be turned to patients with significant cardiac disease
(congestive heart failure, aortic stenosis, severe cardiomyopathy, and/or pulmonary
hypertension), as higher volumes and osmolality of contrast material may result in an
increased risk for a contrast reaction.
 Renal insufficiency: Contrast nephrotoxicity is defined as the rapid deterioration of renal
function after the administration of contrast media, when no other etiology can be
determined from the clinical records. The major predisposing risk factors include
preexisting renal insufficiency (defined as serum creatinine level >1.5 mg/dL) and diabetes.
Other risk factors include dehydration, cardiovascular disease, the use of diuretics,
advanced age (>70 years old), hypertension, and hyperuricemia.  Obtaining multiple
contrast-enhanced studies within a 24-hour period is also thought to increase the risk for
contrast-induced nephrotoxicity.
13
 Miscellaneous: Relative contraindications to the use of high osmolality iodinated contrast
media (HOCM) in patients with pheochromocytoma, sickle cell disease, and multiple
myeloma have been reported. Although the administration of low osmolality or iso-osmotic
contrast media may be beneficial in patients with pheochromocytoma and sickle cell
disease, little evidence suggests that these agents mitigate the risks associated with multiple
myeloma.
As we suspect congenital anomaly of kidneys in this case(congenital aplasia of left kidney and
doubling of the right kidney), according to above mentioned indications there is a most optimal
way to confirm that diagnosis is to prescribe an excretory urography.

2. . A 13-year-old girl complains of fatigability, frequent headaches, cardialgia. Eight years


ago she had a case of pyelonephritis. Urine analyses periodically revealed leukocyturia.
The child has undergone no further treatment.On examination: increased BP up to 150/100
mm Hg. Ultrasound investigation revealed significant reduction of the right kidney. What
process is leading in arterial hypertension pathogenesis in this case?
A. Hyperactivity of renin-angiotensin system
B. Disruption of water-electrolytic balance
C. Disruption of renal circulation
D. Hypersympathicotonia
E. Increased cortisol level
Correct answer is A - Hyperactivity of renin-angiotensin system
Explanation :
Hypertension complicates chronic pyelonephritis. Since arterial narrowing is common in the
damaged kidney, activation of the renin-angiotensin system due to renal ischaemia has been
suggested as a pathogenetic mechanism. Also the renal veins more severely damaged than the
arteries and their lumina were often obliterated by organized thrombus. It suggested that such
widespread obliteration of the renal venous tree could impair blood flow and contribute to the
tissue damage in the pyelonephritic kidney. About 80% of CKD (chronic kidney disease)
patients are hypertensive, and kidney function and blood pressure are clearly related to both
physiologic and pathologic conditions in a "vicious cycle". In this pathologic scenario, there is a
renin-angiotensin system (RAS) hyperactivity associated to progression of renal damage. The
renin-angiotensin-aldosterone system regulates renal vasomotor activity, maintains optimal salt
and water homeostasis, and controls tissue growth in the kidney. However, pathologic
consequences can result from overactivity of this cascade, involving it in the pathophysiology of
kidney disease. An activated renin-angiotensin-aldosterone system promotes both systemic and
glomerular capillary hypertension, which can induce hemodynamic injury to the vascular
endothelium and glomerulus. In addition, direct profibrotic and proinflammatory actions of
angiotensin II and aldosterone may also promote kidney damage. The majority of the untoward
effects associated with angiotensin II appear to be mediated through its binding to the
angiotensin II type 1 receptor. Aldosterone can also induce renal injury by binding to its receptor
in the kidney. An understanding of this system is important to appreciate that inhibitors of this
cascade can reduce the progression of chronic kidney disease in proteinuric disease states.
Pharmacologic agents that can interfere with this cascade include angiotensin-converting enzyme
inhibitors, angiotensin receptor blockers, and aldosterone receptor antagonists.

3. An 8-year-old boy has a 2-year history of blotchy itchy rash appearing after eating ci-
trus fruit. The first eruption occurred at the age of 6 months after the introduction of jui-
ces to the baby’s diet. Father has a history of bronchial asthma,mother - that of allergic rhi-
nitis. What is the most likely diagnosis?
A. Atopic dermatitis
B. Psoriasis
C. Pityriasis Rosea
14
D. Urticaria
E. Quincke’s edema
Correct answer is A: atopic dermatitis
Explanation: Atopic is the most common of the dermatitides seen in infancy and childhood,
but there are numerous other diseases that can mimic the skin findings. These include seborrheic
dermatitis, immunodeficiency, and psoriasis in infancy; scabies, tinea corporis infection,
perioral, nummular, contact, and molluscum dermatitis in childhood. It is sometimes extremely
difficult to differentiate between ichthyosis and AD, and it is also important to differentiate AD
from erythrodermic conditions including acrodermatitis enteropathica, biotin deficiency, and
Netherton syndrome. A rare condition in children that may mimic AD is mycosis fungoides.
Atopic dermatitis (AD) is indistinguishable from other causes of dermatitis. In infancy, the most
common difficulty is distinguishing it from seborrheic dermatitis (SD). This entity is not seen
with the same frequency as a decade ago. Both AD and SD are associated with cradle cap (a
retention hyperkeratosis) found on the vertex of the scalp, which is greasy and yellow in
individuals with SD and dry and crusted in individuals with AD. Other areas of involvement in
SD are the intertriginous areas and diaper area; erythema and a greasy scale can be seen over the
eyebrows and the sides of the nose. In AD, xerosis of the skin and pruritus occur, which are not
usually features of SD. Both conditions should be distinguished from psoriasis. Scabies
manifests in infancy or childhood as a pruritic eruption. Other members of the family are usually
itchy, and the primary sites of involvement are moist, warm areas. The eruption is polymorphic
with a dermatitis, nodules, urticaria, and 6-10 burrows. Pustules on the hands and feet are almost
diagnostic of scabies in infancy. Facial involvement is rare, and xerosis does not occur. Allergic
contact dermatitis from nickel in infants and children is sometimes difficult to distinguish from
AD. A central area of dermatitis on the chest from nickel snaps in undershirts or around the
umbilicus from snaps in jeans is helpful for making the diagnosis, although a dermatitic eruption
may occur as an id reaction in other areas, particularly the antecubital fossae. Xerosis and facial
involvement are absent. AD usually starts earlier than contact dermatitis. Infants with a severe
itch and generalized dermatitis in the setting of recurrent infections should be investigated for
evidence of an immunodeficiency. Failure to thrive and repeated infections help distinguish the
eruption from AD. In Wiskott-Aldrich syndrome, bleeding may be prominent with the
dermatitis, because of the associated thrombocytopenia. A population-based cohort study by
Schmitt J et al. suggested a possible link to the development of mental health issues in patients
who experienced infant eczema and concurrent sleeping problems. In older children, mycosis
fungoides (a form of T-cell lymphoma) often presents with hypopigmented patches associated
with a dermatitis. This entity is being recognized with increased frequency as physicians become
more aware of the disease, and it is sometimes difficult to distinguish between the 2 entities.
Tinea corporis usually manifests as a single lesion, but inappropriate treatment with steroids may
cause a widespread dermatitis. Facial involvement, the presence of xerosis, the age of
appearance, and an early onset (in AD) help distinguish between the two conditions.

Disease/Condition Differentiating Signs/Symptoms Differentiating Tests


Seborrheic Characteristic greasy scale that is Clinical exam is the best tool
dermatitis not pruritic. Often affects cheeks on to differentiate between these
face, scalp, extremities, and trunk. lesions.
Unlike atopic dermatitis, the diaper
area is often affected.
Irritant contact Common in diaper area, face, and Elimination of irritant will
dermatitis extensor surfaces in children result in clinical improvement.
resulting from exposure to irritating Patch testing may be positive
substances. Typically less pruritic for relevant irritant.
than atopic dermatitis. May be limited role for skin
biopsy (e.g., patch tests are
15
negative, chronic disease,
uncertain diagnosis), which
can identify whether contact
dermatitis is present, but does
not distinguish irritant from
allergic contact dermatitis.
Allergic contact Well-circumscribed erythematous Elimination of allergic stimuli
dermatitis lesions, often with spongiotic results in resolution of
papules, vesicles, and crusting. dermatitis.
Lesions are usually pruritic and Patch testing may be positive
asymmetric (unlike those associated for relevant allergen.
with atopic dermatitis, which are May be limited role for skin
typically symmetric). Eruptions are biopsy (e.g., patch tests are
due to contact with specific allergen, negative, chronic disease,
and removal of offending agent uncertain diagnosis), which
results in resolution of symptoms. can identify whether contact
dermatitis is present, but does
not distinguish irritant from
allergic contact dermatitis.
Scabies Severe pruritus, particularly at night. Microscopy may reveal mites,
In addition to papules or vesicles, eggs, or scybala (mite feces).
burrows may be evident and will
help to make the diagnosis. The
wrists, ankles, palms, soles,
interdigital spaces, axilla, waist, and
groin are the most commonly
affected sites. Patients will often
report similar symptoms in family
members or other close contacts.[3]
Psoriasis Well-circumscribed, erythematous This is usually a clinical
lesions with silver scale that show a diagnosis. No laboratory
predilection for extensor surfaces, testing is typically necessary to
particularly elbows and knees. The distinguish between atopic
nail pitting seen in psoriasis has dermatitis and psoriasis. If the
smaller pits and is more common diagnosis is uncertain, there
than that seen in patients with atopic may be a limited role for skin
dermatitis. biopsy, which may not always
reveal the classic features of
psoriasis.
Mycosis fungoides The initial stages of mycosis Skin biopsy is necessary to
fungoides (cutaneous T cell make the diagnosis of mycosis
lymphoma) may look similar to fungoides. Additional studies
atopic dermatitis. Erythematous that encompass cell flow
plaques in random distribution are cytometry may be used to
common and scale is often present. confirm the diagnosis.
As opposed to patients with atopic
dermatitis, patients with mycosis
fungoides tend to be older at the
time of diagnosis, with an average
age of 50 years.

4. An 8-year-old child was hospitalized for fever up to 39, 8oC, inertness, moderate
16
headache, vomiting. Examination revealed meningeal symptoms. Lumbar puncture was
performed. The obtained fluid had raised opening pressure, it was transparent, with
the cell count of 450 cells per 1 mcl (mai-nly lymphocytes - 90%), glucose level of 2,6
mmol/l. What causative agent might have caused the disease in the child?
A. Enterovirus
B. Meningococcus
C. Koch’s bacillus
D. Staphylococcus
E. Pneumococcus
Correctanswer is A: enterovirus
Explanation:
Nonpolio enteroviruses cause an astronomical number of infections per year. More than 90% of
enteroviral infections are either asymptomatic or cause a nonspecific febrile illness. A wide
range of symptoms is observed, but most cases include fever, a viral prodrome, and
gastrointestinal symptoms. Patients with nonspecific febrile illness, the most common form of
enteroviral infection, present with a sudden onset of fever, temperature ranging from 38.5-40°C.
Accompanying symptoms include general upper respiratory and GI complaints. Clinical
indicators include a flulike syndrome consisting of malaise, myalgias, sore throat, headache,
conjunctivitis, nausea, emesis, and diarrhea. Genitourinary manifestations such as orchitis and
epididymitis are possible. Symptoms generally last 3-7 days and are caused by all enteroviral
subtypes. Herpangina occurs in children aged 3-10 years. These patients report painful vesicles
on the posterior pharynx and tonsils. These lesions are associated with fever, sore throat,
odynophagia, and other viral symptoms. Mothers may notice a decreased oral intake by the child
due to the painful ulcers. The causative agent is most commonly coxsackievirus group A and,
sometimes, coxsackievirus group B. Herpangina is self-limited, and symptoms last 3-7 days.
Hand-foot-and-mouth disease is a vesicular eruption in the oropharynx, palms, soles, and
interdigits of toddlers and school-aged children. The oral vesicles are not usually painful.
Patients often present after 1-2 days of fever and have a characteristic viral exanthem. Lesions
are more common on the dorsal surfaces of the hands and feet than in other locations. The most
common causative agent is coxsackievirus group A, serotype 16, but strains of enterovirus 71
circulating in East Asia are currently causing outbreaks of hand-foot-and-mouth disease (HFMD)
that are associated with a serious rhombencephalitis, with significant mortality. Atypical HFM
disease was recently reported around the globe and is caused by coxsackievirus A6. It is
characterized by a relatively paucity of oral lesions, but a striking bullous eruption on the
extremities. Children with eczema may be more affected and "eczema coxsackium" was coined
as far back as 1968 for this condition. [15]Postinfectious loss of the nails is reported frequently.
Viral exanthems, a frequent cause of emergency department visits, manifest as rubelliform or
roseolalike rashes that occur in the summer months. These exanthems occur in children younger
than 5 years and have a benign 3-day to 5-day course. The responsible agents are usually
echoviruses. Patients with aseptic meningitis have symptoms that mimic the initial symptoms of
nonspecific febrile illnesses, but, as aseptic meningitis progresses, patients report a headache,
stiff neck, and photophobia. A nonspecific rash can accompany these symptoms, raising the
question of meningococcemia. The clinical course of aseptic meningitis is self-limited and
resolves in 1-2 weeks. The coxsackievirus group B and echoviruses are responsible for 80-90%
cases in which a causative organism of aseptic meningitis is identified. Neurotropic strains, such
as enterovirus 71, can be responsible for more aggressive cases of CNS infections. Ninety
percent of some cohorts with enterovirus 71 infection also had rhombencephalitis. This can lead
to neurogenic pulmonary edema and has an overall fatality rate of 14%. Early signs of severe
infection include myoclonus and sleep disturbance. Fever that lasts longer than 3 days duration,
high fevers (>38.5C), and lethargy are predictors of CNS involvement. Patients with myocarditis
or pericarditis report chest pain, fatigue, and dyspnea on exertion. These symptoms can progress
to dysrhythmia and heart failure. The most common cause of cardiac involvement is
17
coxsackievirus group B5 infection, but echoviruses are also etiologies of infection. Pleurodynia
(Bornholm disease, devil's grippe) is an uncommon epidemic that causes severe muscular pains
in the chest and abdomen. These sharp pains worsen with breathing or coughing and are
associated with profuse sweating. Spasmodic muscular pains last 15-30 minutes in older children
and adolescents. The condition can mimic serious surgical conditions and can cause periodic
episodes of respiratory difficulty. These symptoms are accompanied by fever, headache,
anorexia, nausea, and emesis. Symptoms last for 2 days. Coxsackie viruses B3 and B5 infect the
intercostals muscles, causing these frightening but rare outbreaks. Neonates with nonpolio
enterovirus infections are at a high risk of developing a sepsislike condition, including
meningoencephalitis, myocarditis, and hepatitis. Presenting symptoms include poor feeding,
lethargy, fever, irritability, hypoperfusion, and jaundice. Differentiating these infections on
clinical grounds from bacterial sepsis is impossible. Infants younger than 10 days are unable to
mount a significant immune response and are at a higher risk of a serious infection from
echoviruses and coxsackie group B viruses. A history of a mother who had a febrile illness with
GI symptoms around the time of birth is often reported; this acute presentation results in
exposure to viral shedding without significant transplacental transfer of maternal antibodies.
Poliovirus infections are divided into 4 groups of clinical syndromes: asymptomatic, abortive,
nonparalytic, and paralytic. Most infections (90-95%) are asymptomatic. Abortive poliomyelitis
involves a nonspecific febrile illness that spares the CNS and spontaneously resolves after a few
days. Temperature is not higher than 103°F. Patients report a minor febrile upper respiratory
infection, such as cough and sore throat, and gastrointestinal infection with nausea and diarrhea.
Patients with nonparalytic poliomyelitis (aseptic meningitis) present in the same manner as
patients with abortive poliovirus, but nonparalytic poliomyelitis progresses to aseptic meningitis.
During the initial flulike illness, patients report stiffness in the posterior neck muscles, limbs, and
trunk. This minor viremia is followed by nuchal and spinal rigidity, the hallmark of nonparalytic
polio. Paralytic poliomyelitis starts with a nonspecific febrile illness and muscle weakness that
resolves after 2-3 days but is followed by a sudden onset of asymmetric flaccid paralysis. Pain,
nuchal rigidity, and hypertonia are indicators of brainstem, spinal ganglia, and posterior column
involvement. Bulbar poliomyelitis involves the speech and central cardiorespiratory centers of
the brain stem and can cause death because of cessation of cardiac and respiratory activity.

5. An 11-year-old boy complains of general weakness, fever up to 38, 2oC, pain and swelli-
ng of the knee joints, sensation of irregular heartbeat. 3 weeks ago, the child had a case
of tonsillitis. Knee joints are swollen, the overlying skin and skin of the knee region
is reddened, local temperature is increased, movements are limited. Heart sounds are
muffled, extrasystole is present, auscultati-on reveals apical systolic murmur that is not
conducted to the left inguinal region. ESR is 38 mm/hour. CRP is 2+, antistreptolysin O ti-
tre - 400.What is the most likely diagnosis?
A. Acute rheumatic fever
B. Vegetative dysfunction
C. Non-rheumatic carditis
D. Juvenile rheumatoid arthritis
E. Reactive arthritis
Correctanswer is A: acute rheumatic fever.
Explanation:
In the final Jones criteria, different diagnostic criteria were established for the diagnosis of acute
rheumatic fever for low risk and moderate-high risk populations. The changes in the diagnostic
criteria for low-risk populations include subclinical carditis found on echocardiogram as a major
criterion in addition to carditis found clinically and a body temperature of 38.5°C and above as a
minor criterion. In moderate-high risk populations, subclinical carditis found on echocardiogram
in addition to clinical carditis is used as a major criterion as a new amendment. In addition,
aseptic monoarthritis and polyarthralgia are used as major criteria in addition to migratory
18
arthritis and monoarhtralgia is used as a minor criterion among joint findings. However,
differentiation of subclinical carditis from physiological valve regurgitation found in healthy
individuals and exclusion of other diseases involving joints when aseptic monoarthritis and
polyarthralgia are used as major criteria are very important. In addition, a body temperature of
38°C and above and an erythrocyte sedimentation rate of 30 mm/h and above have been accepted
as minor criteria. The diagnostic criteria for the first attack have not been changed; three minor
findings have been accepted in presence of previous sterptococcal infection in addition to the old
cirteria for recurrent attacks. In the final Jones criteria, it has been recommended that patients
who do not fully meet the diagnostic criteria of acute rheumatic fever should be treated as acute
rheumatic fever if another diagnosis is not considered and should be followed up with benzathine
penicilin prophylaxis for 12 months. It has been decided that these patients be evaluated 12
months later and a decision for continuation or discontinuation of prophylaxis should be made.
In countries where the disease is prevalent, it is very important for physicians to make an
accurate diagnosis of acute rheumatic fever with their own logic and assessment in addition to
the criteria proposed.
Major criteria
Arthritis:
Arthritis is the most common sign observed in ARF and the least specific one. It is observed in
older patients. In 1992 Jones criteria, migratory polyarthritis was used as a major criterion (1).
The disease frequently involves large joints unilaterally. It frequently involves the knee, elbow,
wrist and ankle. Although the hip and shoulder joints are large joints, they are involved rarely.
Arthritis is migratory and passes from one joint to another. Arthritis in each joint lasts shorter
than two weeks. Migratory arthritis observed in ARF generally resolves spontaneously in a few
days or weeks (in 4 weeks at the latest) even if it is not treated. Arthritis of ARF responds very
well to treatment with salicylates. In case of arthritis lasting longer than 48 hours under salicylate
treatment at an appropriate dose, another diagnosis should be considered.
Erythema marginatum:
This finding is observed rarely. No amendment related with eryhtema marginatum was made in
the final Jones criteria (11). Erythema marginatum is in the form of map-like, painless and
unitchy erythema which becomes pale from the center with irregular borders generally observed
on the trunk and the internal side of the arms and legs and not observed on the face. Erythema
marginatum is migratory like arthritis. It is aggravated when the environmental temperature
incrases and may disappear in cold environment. It spontaneously resolves after a few days.
Although it is a major criterion of ARF, it is not diagnostic alone without the presence of another
major criterion, because it may also be found in other conditions including acute
glomerulonephritis and drug-induced rash.
Subcutaneous nodules:
Subcutaneous nodules are observed rarely. No amendment related with subcutaneous nodeules
was made in the final Jones criteria. Subcutaneous nodules are painless nodules which are not
attached to the surrounding with a size of 0.5–2 cm observed on the extansor surfaces of joints,
scapula or mastoid. They disappear in a few days-weeks. Although they are a major criterion of
ARF, they are not diagnostic alone without the presence of another major criterion, because they
may also be found in other conditions including juvenile idiopathic arthritis and systemic lupus
erythematosus.
Minor criteria:
Fever:
Currently, fever is observed less frequently. While a body temperature of 39 °C and above was
used as a minor criterion in the previous Jones criteria, a body temperature of 38.5°C and above
in low-risk populations and a body temperature of 38 °C and above in moderate- and high-risk
populations were accepted as minor criteria in the final Jones criteria. A body temperature of 38
°C and above should be used as a minor criterion in our country. Fever which is found in the
beginning of the disease decreases in a few days.
19
Arthralgia:
According to the final Jones criteria, polyarthralgia in low-risk populations and monoarthralgia
in moderate- and high-risk populations including our country are used as minor criteria.
However, exclusion of the other causes by differential diagnosis is very important in terms of
making an accurate diagnosis when joint findings are used for the diagnosis. When joint findings
are used as major criteria, they can not be used as minor criteria.
PR interval:
PR prolongation on ECG is observed frequently in patients with ARF. In the final Jones criteria,
a PR interval longer for age was proposed as a minor criterion, but a PR interval longer for age
and heart rate should be used as a minor criterion, because accompanying fever and myocarditis
may lead to tachycardia in these patients. PR prolongation is not a specific finding for the
disease. Prolongation in PR interval may last for a long time after the acute period in a small
number of patients. PR prolongation can not be used as a minor criterion in patients who have
had carditis.
Laboratory findings indicating acute infection:
Erythrocyte sedimentation rate and C-reactive protein (CRP) are used. An ESR above 60 mm/h
in low-risk populations and an erythrocyte sedimentation rate above 30 mm/h in moderate- and
high-risk populations is used as a minor criterion.
Both in low-risk and moderate- and high-risk populations, a CRP value above 3 mg/L or above
the upper normal limit of the laboratory is used as a minor criterion.

6. A 7-year-old child became ill again 2 weeks after a case of tonsillitis. There are the
following complaints: temperature rise up to 38oC, hemorrhagic rash on the extremities,
enlargement of the ankle joints. Blood test: hemoglobin is 120 g/l, platelets are 170 · 109/l,
ESR is 30 mm/hour. Urine test: proteinuria up to 0,7 g/l, cylinders - 5-6 in the field of vi-
sion, erythrocytes - 8-10 in the field of vision. What mechanism of hemorrhagic syndrome is
present in the given case?
A. Vessel wall damage caused by immune complexes
B. Platelet dysfunction
C. Suppression of hematopoietic stem cells
D. Decrease of adhesive-aggregative function of platelets
E. Vessel wall damage caused by bacteria
Correct answer is A: Vessel wall damage caused by immune
complexes.
Eхplanation : Schönlein-Henoch disease is an immunocomplex disease. Suspected though not
proved inciting agents (antigens) include: group A β-hemolytic streptococci and other bacteria,
viruses, drugs, foods, insect bites. Pathogenesis: antigen influence → immunoglobulin G, M, A
hyperproduction → antigen-antibody-complement complex in the blood → skin, kidney,
intestine, joints precipitation → lesion → new autoantigens production → autoimmune damage
of small vessels.
Laboratory findings:
1. CBC reveals normochromic normocytic anemia of chronic disease; leukocytosis and
thrombocytosis are associated with inflammatory process.
2. The Westergren sedimentation rate is elevated.
3. The platelet count, platelet function test, and bleeding time are usually normal.
4. Blood coagulation studies are normal.
5. Urinalysis frequency reveals hematuria, proteinuria, but casts are uncommon.
6. The ASO (antistreptolizin-O) titer is frequently elevated and the throat culture positive for
group A beta-hemolytic Streptococci.
7. Serum Ig A may be elevated.
Histologic Findings: Leukocytoclastic vasculitis observed in Henoch-Schцnlein purpura, is
characterized by focal segmental necrotizing full-thickness lesions of varying stages in small
20
vessels. Fibrinoid necrosis is present. The cellular infiltrate is predominantly polymorphonuclear
neutrophils. Lymphocytes and eosinophils may be present. Henoch-Schцnlein purpura reveals a
leukocytoclastic vasculitis with IgA immune deposits.

7. A 4-month-old boy has been undergoing in-patient treatment for pneumocystic


pneumonia for 4 weeks. The diagnosis has been made based on clinical signs, typi-
cal X-ray presentation, presence of severe hypoxemia, positive dynamics caused by
intravenous introduction of Biseptol (Cotrimoxazole). Anamnesis states that enzyme-
linked immuno sorbent assay (ELISA) detected antibodies to HIV in the umbilical
blood. Polymerase chain reaction (PCR) was performed on the child at the ages of 1 month
and 3months, and proviralDNAwas detected in the child’s blood. Viral load and number of
CD4+-lymphocytes was not measured. Make the diagnosis:
A. HIV/AIDS
B. Infectious mononucleosis
C. Pneumonia
D. Tuberculosis
E. Adenovirus infection
Correct answer is A: HIV/AIDS.
Explanaion:
Signs and symptoms of pediatric HIV infection include the following:
 Unusually frequent and severe occurrences of common childhood bacterial infections, such
as otitis media, sinusitis, and pneumonia
 Recurrent fungal infections, such as candidiasis (thrush), that do not respond to standard
antifungal agents: Suggests lymphocytic dysfunction
 Recurrent or unusually severe viral infections, such as recurrent or disseminated herpes
simplex or zoster infection or cytomegalovirus (CMV) retinitis; seen with moderate to
severe cellular immune deficiency
 Growth failure
 Failure to thrive
 Wasting
 Failure to attain typical milestones: Suggests a developmental delay; such delays,
particularly impairment in the development of expressive language, may indicate HIV
encephalopathy
 Behavioral abnormalities (in older children), such as loss of concentration and memory,
may also indicate HIV encephalopathy

Signs and symptoms of pediatric HIV infection found during physical examination include the
following:
 Candidiasis: Most common oral and mucocutaneous presentation of HIV infection
 Thrush in the oral cavity and posterior pharynx: Observed in approximately 30% of HIV-
infected children
 Linear gingival erythema and median rhomboid glossitis
 Oral hairy leukoplakia
 Parotid enlargement and recurrent aphthous ulcers
 Herpetic infection with herpes simplex virus (HSV): May manifest as herpes labialis,
gingivostomatitis, esophagitis, or chronic erosive, vesicular, and vegetating skin lesions;
the involved areas of the lips, mouth, tongue, and esophagus are ulcerated
 HIV dermatitis: An erythematous, papular rash; observed in about 25% of children with
HIV infection
 Dermatophytosis: Manifesting as an aggressive tinea capitis, corporis, versicolor, or
onychomycosis

21
 Pneumocystis jiroveci (formerly P carinii) pneumonia (PCP): Most commonly manifests as
cough, dyspnea, tachypnea, and fever
 Lipodystrophy: Presentations include peripheral lipoatrophy, truncal lipohypertrophy, and
combined versions of these presentations; a more severe presentation occurs at puberty
 Digital clubbing: As a result of chronic lung disease
 Pitting or nonpitting edema in the extremities
 Generalized cervical, axillary, or inguinal lymphadenopathy

Within the first 48 hours, 14 days, and 4 weeks of life, 38%, 93%, and 96% of infected children,
respectively, have positive HIV DNA PCR results. Any positive HIV DNA PCR finding should
be confirmed with follow-up HIV DNA PCR before infection is diagnosed. Monitor CD4 + levels
or percentages in infants or patients newly diagnosed with HIV at 3- to 4-month intervals to
assess patients' immune status. In children younger than 5 years, the 2010 Panel recommends
using CD4 percentages over absolute CD4 counts for monitoring disease progression because of
inherent age-related changes in absolute CD4 counts.

8. A 5-year-old girl was hospitalized with complaints of pain and swelling in the right
knee joint, temperature rise up to 38, 4oC and a rash diagnosed as erythema annulare centri-
fugum. The signs developed 3 days after the recovery from a case of acute respiratory di-
sease. Name the etiotropic drug to be prescribed:
A. Augmentin
B. Methotrexate
C. Metypred (Methylprednisolone)
D. Diclofenac sodium
E. Captopril
Correct answer is A: Augmentin
Explanation :
Antibiotic treatment in patients who present with acute rheumatic fever (ARF) is necessary
irrespective of the throat culture result. Such therapy probably does not alter the risk of
developing rheumatic heart disease but at least minimizes the possible transmission of a
rheumatogenic streptococcal strain.  For  primary (decrease likelihood of rheumatic fever by
treatment of GABHS pharyngitis) as well as secondary (prevention of recurrent rheumatic
fever/carditis) prophylaxis of rheumatic fever. Although amoxicillin (augmentin) may be used
instead, there is no microbiologic advantage. Do not use tetracyclines to treat GABHS
pharyngitis. For recurrent GABHS pharyngitis, a second 10-d course of the same antibiotic may
be repeated. Alternate drugs (penicillin allergic patients) include azithromycin, clarithromycin or
clindamycin.
Doses:
Mild to moderate infections
 <3 months: ≤30 mg/kg/day PO divided q12hr for 48-72 hours; for ≥10 days for S pyogenes
infections
 >3 months and <40 kg: 25 mg/kg/day PO divided q12hr or 20 mg/kg/day PO divided q8hr 
 >40 kg: 500 mg PO q12hr or 250 mg PO q8hr for 10-14 days
Severe infections
 <3 months: ≤30 mg/kg/day PO divided q12hr for 48-72 hours; for ≥10 days for S pyogenes
infections
 >3 months and <40 kg: 45 mg/kg/day PO divided q12hr or 40 mg/kg/day PO divided q8hr  
 >40 kg: 875 mg PO q12hr or 500 mg PO q8hr for 10-14 days
Tonsillitis/Streptococcal pharyngitis
 50 mg/kg PO qDay for 10 days, not to exceed 1 g/day, OR 25 mg/kg PO BID for 10 days,
not to exceed 500 mg/dose 

22
 >12 years: 775 mg (Moxatag) PO qDay for 10 days, taken within 1 hour after meal
(swallow tablet whole; do not crush or chew)

9. During medical examination of a group of children under 4 years carried out by a


pediatric team in one of the African countries a set of similar pathological signs was
detected in some of the children. The signs are as follows: growth inhibition, mental
changes, muscle atrophy, swellings, changes in hair and skin pigmentation. These children
were diagnosed with kwashiorkor. What food products should be added to the diet to treat
this disorder?
A. Fish, vegetables, cereals
B. Milk, meat, vegetables
C. Vegetables, fruit
D. Cereals, fruit, berries
E. Poultry, fruit, berries
Correct answer is A: Fish, vegetables, cereals
Explanation:
Diet for treating Kwashiorkor
Because people with kwashiorkor have been deprived of a nutritionally adequate diet for a long
time, a medical professional should monitor and plan their food regimen. Treatment should start
with a gradual introduction of carbohydrate foods such as fruits, starchy vegetables, breads and
cereals to provide calories. Then the persons should consume foods containing proteins such as
meat, fish, poultry, eggs, soybeans and legumes. Milk and milk products are also rich in protein.
However, children suffering from kwashiorkor may be lactose-intolerant and may need lactase
enzyme supplements to digest milk, yogurt and cheese.
Give high energy and high protein diet.
. Start with light food which is high in protein and energy:
- Give water drained from boiled green grams, Sugar cane juice, Fruit juices, Vegetables soups,
Porridge of lentil and rice, Porridge of pulses and cereals, Milk, milk shakes.
- Make sure food is cooked properly.
. Continue breastfeeding if possible.
. Along with breast feeding give appropriate and adequate complementary feeding, in weaning
stage -
- Give boiled and mashed vegetables and fruits, porridge of pulses and cereals, water drained
from boiled green grams, Sugar cane juice Fruit juices.
. Wide variety of food should be selected to ensure all vitamins and minerals are included in the
diet.
. Increase intake of antioxidants like vitamin A: all yellow fruits and vegetables contain Vitamin
A. Meat, eggs, fish, kidney and liver, liver oils of fish like cod, shark, and halibut are richest
source of vitamin A.
. Include Vitamin C, antioxidant:
- Citrus fruits vegetable soups and vegetables- cabbage, broccoli, tubers- potatoes and sweet
potatoes
. Increase intake of vitamin E: foods of animal origin are low in vitamin E.
- Oilseeds and vegetable oils like soy, cotton-seed and safflower oil.
- Wheat germ, whole grain, corns, cereals, pulses, nuts, dark green leafy vegetables, olives.
. Another very good antioxidant selenium should be included in the diet: animal foods like - fish,
shellfish, eggs, red meat, chicken and liver.
- Grains, Brazil nuts, garlic, wheat germ and brewer's yeast are good source of selenium.
. Include Japanese sea vegetables (arame, hijiki, kombu, nori) in diet if available, they are
remarkably nutritious.

23
. Wheat grass juice is recommended - grow wheat grains in a pot - grass comes up in a weeks
time so its advisable to plant seven pots so as to have every days supply. Extract juice from these
grass and consume twice a day. It helps you to develop immunity and is highly nutritious.
. Drink boiled mixture of water, mint and basil leaves.
. Increase consumption of zinc, decreases diarrhea episodes:
- seafood (especially oysters), beef, oatmeal, chicken, liver, milk, spinach, ,sea plants nuts and
seeds,
- Plant foods are low in zinc, Whole wheat grains provide good amount of zinc.
. Add a pinch of turmeric to food; it will boost up your immunity.

10. An 8-year-old girl was diagnosed with signs of Morgagni-Adams-Stokes disease


that developed against the background of the III degree atrioventricular heart block. What
drug should be introduced intravenously for emergency aid?
A. Atropine
B. Prednisolone
C. Dobutamine
D. Digoxin
E. Potassium chloride
Сorrect answer is A: Atropine
Explanation:
Emergency care for recurrences of the asystolic form of MAC attacks begins with a precordial
stroke (in children of an early age it is not recommended to apply), then intravenously inject
0.1% atropine solution from the calculation of 10-15 μg / kg or 0.5% isoprenaline solution in a
dose from 0.1-1 μg / kghmin) to 3-4 μg / kghmin), and at the older age - 2-10 μg / kghmin).
Atropine can be re-injected every 3-5 minutes (depending on the effect) to achieve a total dose of
40 mcg / kg (0.04 mg / kg). With insufficient effectiveness of drug treatment, transesophageal,
external percutaneous or intravenous electrostimulation of the heart is performed under
electrocardiographic control.

11. On the 9th day after childbirth the obstetric patient developed high fever up to 38oC. She
complains of pain in the right mammary gland. The examination revealed the following: a
sharply painful infiltrate can be palpated in the right mammary gland, the skin over the infiltrate
is red, subareolar area and nipple are swollen and painful. What is your diagnosis?
A. Abscess of the right mammary gland
B. Mastopathy
C. Cancer of the right mammary gland
D. Serous mastitis
E. Fibrous cystic degeneration of the right
mammary gland
Сorrect answer is A: Abscess of the right mammary gland
Explanation:
Speaking about physiologic engorgement in newborn infants , we must understand that this
pathology is found in 75% of all children. Most often it is of course girls, but 50% of newborn
boys exposed to such processes. Therefore you should not panic in the presence of such things,
you just need to consult a doctor and make sure all is well. But the symptoms of diseases, which
can also be at this age need to know clearly and differential diagnosis. The first symptoms that
should alert, is the emergence of child anxiety, crying or refusal of the breast. If the child was
breast engorgement and everything was fine, but then he began to lose weight or give up the
breast with a shriek, then you need to think about the disease. Often the disease develops at the
end of the first month of life, when the physiological swelling should go, then it may join the
infection and develop mastitis. Mastitis- an inflammation of the mammary gland in the baby, it is
equally the case for girls and boys. Given the same anatomical features of the breast of a young
24
child, loose connective tissue contributes to inflammation of instant depth, so the condition of
the child in the development of mastitis instantly deteriorates. The child is not inclined to limit
the inflammatory process as inclined adult. Therefore, the main symptom of breast newborn
inflammatory disease is very high temperature. It rises very sharply at a mastitis and may be
accompanied by convulsions. On examination, breast mom can see not only increasing it, but the
skin changes, redness or even blue. If you try to baby the breast, it instantly reacts as it is
accompanied by severe pain. If an abscess, it can feel like pus moves under your fingers during
palpation. This process is usually one-sided. Allocation may also be at the same time from the
nipple on the affected side as a green or yellow pus. But we should not kid much torment and
check all the symptoms, if in any doubt best to consult a doctor. Therefore, the main diagnostic
feature is not physiological state, and the newborn breast disease is a change in the child's
condition.

12. A 16-year-old girl addressed a doctor with complaints of fatigability and dizziness. On
heart auscultation: systolic murmur in the II intercostal area along the breastbone edge
on the left. ECG revealed signs of the right ventricular hypertrophy. X-ray revealed di-
latation of the the pulmonary artery trunk, enlargement of the right heart.What heart di-
sorder is it?
A. Pulmonary artery outflow stenosis
B. Fallot’s tetrad
C. Defect of the interatrial septum
D. Coarctation of aorta
E. Pulmonary artery valve failure
Сorrect answer is A: Pulmonary artery outflow stenosis
Many children with pulmonic stenosis remain asymptomatic for years and do not present to a
physician until adulthood. Even then many patients remain asymptomatic. When symptoms of
pulmonic stenosis develop, they resemble those of aortic stenosis (syncope, angina,
dyspnea).Visible and palpable signs reflect the effects of right ventricular (RV) hypertrophy and
include a prominent jugular venous a wave (due to forceful atrial contraction against a
hypertrophied RV), an RV precordial lift or heave, and a left parasternal systolic thrill at the 2nd
intercostal space.
Auscultation: On auscultation, the 1st heart sound (S 1) is normal and the normal splitting of
the 2nd heart sound (S2) is widened because of prolonged pulmonic ejection (P 2, the pulmonic
component of S2, is delayed). In RV failure and hypertrophy, the 3rd and 4th heart sounds
(S3 and S4) are rarely audible at the left parasternal 4th intercostal space. A click in congenital PS
is thought to result from abnormal ventricular wall tension. The click occurs early in systole
(very near S1) and is not affected by hemodynamic changes. A harsh crescendo-decrescendo
ejection murmur is audible and is heard best at the left parasternal 2nd (valvular stenosis) or 4th
(infundibular stenosis) intercostal space with the diaphragm of the stethoscope when the patient
leans forward. Unlike the aortic stenosis murmur, a pulmonic stenosis murmur does not radiate,
and the crescendo component lengthens as stenosis progresses. The murmur grows louder
immediately with Valsalva release and with inspiration; the patient may need to be standing for
this effect to be heard.
Diagnosis

 Echocardiography
Diagnosis of pulmonic stenosis is confirmed by Doppler echocardiography, which can
characterize the severity as
 Mild: Peak gradient < 36 mm Hg
 Moderate: Peak gradient 36 to 64 mm Hg
 Severe: Peak gradient > 64 mm Hg

25
ECG may be normal or show RV hypertrophy or right bundle branch block. Right heart
catheterization is indicated only when 2 levels of obstruction are suspected (valvular and
infundibular), when clinical and echocardiographic findings differ, or before intervention is
done.

13. Posture of an 11-year-old boy was determined during preventive examination. The child
presents with curled forward rounded shoulders, the head is bowed forward, the thorax is
flattened, the stomach is bulging. In the vertebral column there are deepened cervical and lumbar
flexures. What posture does the child have?
A. Kyphosis
B. Lordosis
C. Stooping
D. Corrected
E. Normal
Correct answer is A :
The following are the most common symptoms of kyphosis. However, each child may
experience symptoms differently. Symptoms may include:
 Difference in shoulder height
 The head bends forward compared to the rest of the body
 Difference in shoulder blade height or position
 When bending forward, the height of the upper back appears higher than normal
 Tight hamstrings (back thigh) muscles
Back pain, pain down the legs, and changes in bowel and bladder habits are not commonly
associated with kyphosis. A child experiencing these types of symptoms requires immediate
medical evaluation.
The symptoms of kyphosis may resemble other spinal conditions or deformities, or may be a
result of an injury or infection.

14. The left hand of a newborn is extended in all its joints, stretched along the torso, and
pronated in the forearm. Active movements of the shoulder joint are retained. The hand
is flattened, atrophied, cold to touch, hangs passively. Grasping and Babkin’s reflexes are
absent at the affected side. Hemogram indicators are normal. Make the most likely diagnosis:
A. Inferior distal obstetrical paralysis
B. Osteomyelitis
C. Proximal obstetrical paralysis
D. Complete obstetrical paralysis
E. Hypoxic-ischemic encephalopathy
Correct answer is A: Inferior distal obstetrical paralysis
Klumpke’s palsy affects the hand’s intrinsic muscles and the flexors of the wrist and fingers.
Depending on the type of injury to the brachial plexus, symptoms vary from mild to severe. The
most severe physical  manifestation of Klumpke’s palsy is the “claw hand,” where the affected
forearm tends to lie flat and the wrist and fingers are tightened.
Other symptoms of Klumpke’s palsy include:
Severe pain
C8/T1 Dermatome distribution numbness
Weakness or lack of ability to use specific muscles of the shoulder, arm. or hand.
Limp or paralyzed arm
Stiff joints
Atrophy of the muscles
No feeling or sensation in the affected arm or hand
Drooping of the eyelids on the opposite side of the face (ptosis and miosis), also known as
Horner’s syndrome
26
15. A 14-year-old girl has been delivered to a gynecological department with complaints
of profuse blood discharge from her genital tract for 2 weeks.Anamnesis:menstruation since 13,
irregular, painful, profuse; the last one was 2 months ago. Objectively: pale skin and
mucosa, BP- 100/60 mm Hg, Hb- 108 g/l. The abdomen is soft and painless on palpation.
Rectal examination revealed no pathologies of reproductive organs.What condition is it?
A. Juvenile uterine hemorrhage (Dysfunctional)
B. Hypomenstrual syndrome
C. Inflammation of uterine appendages (Pelvic inflammatory disease)
D. Pelviperitonitis
E. Endometritis
Correct answer is A: Juvenile uterine hemorrhage (Dysfunctional)
Dysfunctional uterine bleeding (DUB) is abnormal genital tract bleedingbased in the uterus and
found in the absence of demonstrable structural or organic disease. It is usually due to
hormonal disturbances: reduced levels of progesterone cause low levels of prostaglandin
F2alpha and causemenorrhagia (abnormally heavy flow); increased levels of tissue plasminogen
activator (TPA) (a fibrinolytic enzyme) lead to more fibrinolysis. Diagnosis must be made by
exclusion, since organic pathology must first be ruled out. DUB can be classified as ovulatory or
anovulatory, depending on whetherovulation is occurring or not. It is usually a menstrual
disorder, although abnormal bleeding from the uterus is possible outside menstruation. Some
sources state that the term "dysfunctional" implies a hormonal mechanism. Use of the term
"abnormal uterine bleeding" is preferred in today's medicine.

16. A 1,5-month-old child on breasfeeding presents from birth with daily vomiting,
irregular liquid foamy feces, and meteorism, which are resistant to antibacterial and
probiotic therapy; no increase of body mass is observed. The child’s condition improved,
when breastmilk was substituted with ”NAN low lactose” formula. What pathology is it?
A. Lactase deficiency
B. Intestinal lambliasis (Giardiasis)
C. Infectious enteritis
D. Drug-induced enteritis
E. Functional dyspepsia

Corrrect answer is A: Lactase deficiency


Explanation:
 Primary lactase deficiency is attributable to relative or absolute absence of lactase that
develops in childhood at various ages in different racial groups and is the most common cause
of lactose malabsorption and lactose intolerance. Primary lactase deficiency is also referred to
as adult-type hypolactasia, lactase nonpersistence, or hereditary lactase deficiency.
 Secondary lactase deficiency is lactase deficiency that results from small bowel injury, such as
acute gastroenteritis, persistent diarrhea, small bowel overgrowth, cancer chemotherapy, or
other causes of injury to the small intestinal mucosa, and can present at any age but is more
common in infancy.
Congenital lactase deficiency is a rare disorder that has been reported in only a few infants.
Affected newborn infants present with intractable diarrhea as soon as human milk or lactose-
containing formula is introduced. Small intestinal biopsies reveal normal histologic
characteristics but low or completely absent lactase concentrations. Unless this is recognized
and treated quickly, the condition is life-threatening because of dehydration and electrolyte
losses. Treatment is simply removal and substitution of lactose from the diet with a
commercial lactose-free formula.

27
Bloating, abdominal discomfort, meteorism, and flatulence that occur from 1 hour to a
few hours after ingestion of milk or dairy products may signify lactose intolerance; however,
other disorders such as milk-protein sensitivity, allergic-type reactions to other substances in
the meal, or other saccharide intolerance may cause similar symptoms.

Physical examination may reveal the following:


 Abdominal pain: Nonspecific, nonfocal abdominal pain and cramping are common and are
sometimes associated with bloating and flatus. This pain may mildly increase with
palpation. Focal abdominal pain significantly worsened by palpation, the presence of
rebound tenderness, or guarding should alert the clinician to a more serious and possibly
surgical GI diagnosis.
 Borborygmi: A significant increase in peristaltic activity in the small bowel can cause an
audible or palpable increase in bowel activity.

17. A 2-year-old child has been suffering since birth from recurring inflammatory diseases of
lungs, purulent pansinusites, hearing deterioration, multiple cylindrical bronchiectases.
Dextrocardia is observed. On biopsy: ultrastructural change of ciliated epithelium. What is the
basis of the given syndrome?
A. Primary ciliary dyskinesia
B. Proteoglycans insufficiency
C. Surfactant deficit
D. Muscle cells atony
E. Alpha-1-antitrypsin deficiency

Correct answer is A - Primary ciliary dyskinesia.


Primary ciliary dyskinesia (PCD) is predominantly inherited as an autosomal recessive
disorder leading to recurrent and chronic upper and lower respiratory tract infection, and, in 40–
50% of cases, mirror-image organ arrangement and other forms of heterotaxy. Children with
PCD often have a clinical history of lower airway disease, manifested in a chronic wet-sounding
cough and, occasionally, wheeze or shortness of breath. In addition, virtually all subjects show
evidence of chronic upper airway symptoms, such as chronic rhinitis (nasal discharge, episodic
facial pain and anosmia). This may be confirmed by physical examination and/or sinus imaging.
Ear symptoms (recurrent otitis media and glue ear) are a frequent complication that can require
multiple interventions, including repeated courses of antibiotics. Diagnosis of PCD is frequently
delayed, in part because patients present with symptoms (rhinitis, secretory otitis media, cough
and recurrent bronchitis) that are common in healthy children.

18. A newborn infant (the first labor, lasted for 26 hours) is 1-day-old, postmature; body weight
is 3850 g; body length is 52 cm. Delivery was performed by applying obstetrical forceps in
sincipital presentation, Apgar score is 1/3. The face is bluish-pale. The head is thrown back;
severe birth trauma is present; the infant is excitable, shrill "cerebral scream"is present; the eyes
are half-open; facial expression is attentive; hyperesthesia, hypersthenia and readiness for
convulsions are present. Liquor has high content of erythrocytes, lymphocytic cytosis occurs.
The most likely diagnosis is:
A. Subarachnoid hemorrhage
B. Epidural hemorrhage
C. Subdural hemorrhage
D. Intraventricular hemorrhage
E. Intracerebral hemorrhage

Correct answer is A: Subarachnoid hemorrhage

28
The most common symptoms of a subarachnoid hematoma (SAH) are headache,
confusion, vomiting, seizures, and loss of consciousness. SAH also leads to rigidity in the neck
and photophobia. If the SAH continues untreated, a further symptom includes ipsilateral
pupillary dilation, which is an indication of elevated intracranial pressure and brain herniation.
Neurologic symptoms associated with SAH include partial paralysis, loss of vision or double
vision and seizures.

19. A 14-year-old girl complains of tooth caries; the tooth should be filled. Anamnesis states that
a
crtificial mitral valve was installed 2 years ago due to mitral insifficiency. What antibacterial
drug should be prescribed to prevent infective endocarditis?
A. Amoxicillin
B. Lincomycin
C. Ceftriaxone
D. Erythromycin
E. Midecamycin

Correct answer is A: Amoxicillin

Prophylactic (preventive) antibiotics should be given to people who are most likely to have
severe complications if they develop infective endocarditis. In such people, prophylactic
antibiotics are recommended before dental procedures, which have the highest risk of causing
bacteria to enter the blood. There is no evidence to recommend using antibiotics before
procedures on the gastrointestinal or urinary tracts.
Antibiotic prophylaxis should be considered in people with
Prosthetic heart valves
Past episodes of infective endocarditis
Heart transplants who develop disease in their heart valves
Congenital (present since birth) heart diseases who have had prosthetic materials or devices
placed in their heart within the last 6 months
Congenital heart diseases whose heart defects have not been completely fixed or have remaining
defects at or near where a prosthetic patch or device was placed
Antibiotic use
For patients who require prophylactic antibiotics, these medications ideally should be given
before a dental procedure. If this dose is missed, patients can receive antibiotics up to 2 hours
after the procedure.
The most commonly prescribed antibiotic is oral penicillin. People who are allergic to penicillin
can be given cephalosporins, macrolides(different classes of antibiotics), or clindamycin. There
are injectable forms of these antibiotics for people who cannot take medication by mouth.

20. A prematurely born girl is now 8 months old. She has dyspnea, tachycardia,
hepatosplenomegaly, physical developmental lag, limb cyanosis. There is parasternal cardiac
hump, auscultation revealed systolodiastolic murmur in the II intercostal space on the
left. BP is 90/0 mm Hg. What disease can be suspected?

A. Patent ductus arteriosus


B. Coarctation of aorta
C. Stenosis of aortal valve
D. Stenosis of pulmonary artery
E. Nonclosure of interventricular septum

29
Correct answer is A: Patent ductus arteriosus
The clinical history of patients with PDA varies from those who are completely
asymptomatic to those with severe congestive heart failure or Eisenmenger’s syndrome. A small
patent ductus arteriosus often doesn't cause problems and might never need treatment. However,
a large patent ductus arteriosus left untreated can allow poorly oxygenated blood to flow in the
wrong direction, weakening the heart muscle and causing heart failure and other complications.

21. Three days ago a boy underwent removal of a foreign body from under a nail plate. 2
days later he deeloped acute pulsating pain at the end of the nail bone which aggravated at
pressing. Nail fold became hyperemic, body temperature rose up to 37,5o C, there was a change
in nail plate colour. What is the most likely diagnosis?

A. Subungual panaritium
B. Erysipelas
C. Paronychia
D. Erysipeloid
E. Abscess

Correct answer is A:- Subungual panaritium

The affected area in subungual panaritium is soft tissue that is located under the nail. More
often than not, the subungual panaritium develops after a splinter has got under the plate and was
untimely pulled out. Or, on the contrary, they pulled it out, but they did not completely disinfect
the wound. Panaritium in humans arises from the penetration through the damaged skin of
infectious agents. The most common ailment is caused by the following pathogens:
 Staphylococci
 Pseudomonas aeruginosa
 Streptococci
 Enterococci
 fungal microorganisms
 E. coli.

22. A 3-month-old girl has rhinitis, dyspnea, dry cough. She has been sick for 2 days.
Objectively: pale skin, acrocyanosis, hypopnoe; breathing rate is 80/min; over the whole
pulmonary surface there is vesiculotympanitic (bandbox) resonance observed with numerous
bubbling rales (crackles). The most likely diagnosis is:
A. Acute bronchiolitis
B. Pneumonia
C. Mucoviscidosis
D. Foreign body in airways
E. Acute bronchitis

Correct anser is A – Acute bronchiolitis


Explanation :

23. Bronchiolitis is a disorder commonly caused by viral lower respiratory tract infection in
infants from 1 month through 23 months of age. Bronchiolitis is characterized by acute
inflammation, edema, and necrosis of epithelial cells lining small airways, and increased mucus
production. Signs and symptoms typically begin with rhinitis and cough, which may progress to
tachypnea, wheezing, rales, use of accessory muscles, and/or nasal flaring
A. Acute bronchiolitis

30
B. Pneumonia
C. Mucoviscidosis
D. Foreign body in airways
E. Acute bronchitis

Correct answer is A: Acute bronchiolitis

Acute bronchitis begins as a respiratory tract infection that manifests as the common cold.
Symptoms often include coryza, malaise, chills, slight fever, sore throat, and back and muscle
pain.
The cough in these children is usually accompanied by a nasal discharge. The discharge is
watery at first, then after several days becomes thicker and colored or opaque. It then becomes
clear again and has a mucoid watery consistency before it spontaneously resolves within 7-10
days. Purulent nasal discharge is common with viral respiratory pathogens and, by itself, does
not imply bacterial infection.
Initially, the cough is dry and may be harsh or raspy sounding. The cough then loosens and
becomes productive. Children younger than 5 years rarely expectorate. In this age group, sputum
is usually seen in vomitus (ie, posttussive emesis). Parents frequently note a rattling sound in the
chest. Hemoptysis, a burning discomfort in the chest, and dyspnea may be present.

24. A 14-year-old girl has fainted during a meeting. The day before she complained of a
headache. The skin is pale, the limbs are cold, shallow breathing, heart sounds are muffled; heart
rate is 51/min.; BP is 90/50 mm Hg. The abdomen is soft. Meningeal symptoms are negative.
Make the provisional diagnosis:
A. Collapse
B. Unconsciousness
C. Acute left ventricular failure
D. Acute right ventricular failure
E. Respiratory failure

Correct answer is A: Collapse


Collapse in children is most often neurally mediated and usually has a natural history of
spontaneous resolution or improvement. Syncope is defined as the temporary loss of
consciousness resulting from a reversible disturbance of cerebral function. It is characterized by
a loss of consciousness due to a lack of cerebral blood flow, rapid or sudden onset, falling of the
patient, if not supported, and transiency of the attack. In children, it is most often benign, but
may sometimes herald a more serious, potentially life-threatening cause.

25. A 4-month-old child was admitted to a surgical department 8 hours after the first attack of
anxiety. The attacks happen every 10 minutes and last for 2-3 minutes, vomiting occurred once.
Objectively: the child’s condition is grave. Abdomen is soft, palpation reveals a tumor-like
formation in the right iliac area. After rectal examination the doctor’s finger was stained with
blood. What is the most probable diagnosis?
A. Ileocecal invagination
B. Gastrointestinal haemorrhage
C. Wilm’s tumour
D. Helminthic invasion
E. Pylorostenosis

Correct anser is: A Ileocecal invagination

31
Explanation :
In ileocecal invagination, a segment of the terminal ileum is retained at the level of the
ileocecal valve. However, it may be difficult to differentiate between ileocecal and ileocolic
invagination intraoperatively. The classic clinical triad of acute abdominal pain (colic), currant-
jelly stools or hematochezia, and a palpable abdominal mass is present in less than 50% of
children with intussusception. The onset of nonspecific abdominal symptoms in which vomiting
predominates, the absence of passage of blood via the rectum (usually in cases of less than 48
hours duration), and the inability to obtain a reliable history from these nonverbal children lead
to dismissal of the diagnosis of intussusception in almost 50% of cases. The imaging modalities
most frequently used in the diagnosis of invagination include plain or contrast-enhanced
radiograph, abdominal ultrasound, and computer tomography.

26. A 5-year-old child has body temperature risen up to febrile numbers, suffers from inertness,
weakness. Examination revealed hemorrhage on the skin of limbs and torso. Enlargement of
cervical and axillary lymph nodes can be detected. The liver is 4 cm below the costal arch; the
spleen is 6 cm below the costal arch. Blood test: erythrocytes - 2, 3 · 1012/l, Hb- 60 g/l, platelets
- 40 · 109/l, leukocytes - 32, 8 · 109/l, eosinophiles - 1%, band neutrophiles - 1%, segmented
neutrophiles - 12%, lymphocytes - 46%, monocytes - 1%, blasts - 40%, Duke’s bleeding time
test result is 9 min. What examination is necessary to make the diagnosis?
A. Myelogram
B. Lymph nodes biopsy
C. US of abdominal cavity
D. Detection of hepatitis markers
E. Investigation of platelets dynamic functions

Correct anser is A: Myelogram

Explanation :
In this case, it is necessary to exclude acute leukemia. The leukemias are the most
common malignant neoplasms in childhood. The leukemias may be defined as a group of
malignant diseases in which genetic abnormalities in a hematopoietic cell give rise to an
unregulated clonal proliferation of cells. The result is a disruption of normal marrow function
and, ultimately, marrow failure. The clinical features, laboratory findings, and responses to
therapy vary depending on the type of leukemia. The initial presentation of leukemias usually is
nonspecific and relatively brief. Anorexia, fatigue, and irritability often are present, as is an
intermittent, low-grade fever. Bone or, less often, joint pain, particularly in the lower extremities,
may be present. Patients often have a history of an upper respiratory tract infection in the
preceding 1–2 mo. Less commonly, symptoms may be of several months' duration, may be
localized predominantly to the bones or joints, and may include joint swelling. Bone pain is
severe and may wake the patient at night. As the disease progresses, signs and symptoms of bone
marrow failure become more obvious with the occurrence of pallor, fatigue, bruising, or
epistaxis, as well as fever, which may be caused by infection.
The diagnosis of acute leukemia is strongly suggested by peripheral blood findings
indicative of bone marrow failure. Anemia and thrombocytopenia are seen in most patients.
Leukemic cells often are not observed in the peripheral blood in routine laboratory examinations.
Many patients with acute leukemia present with total leukocyte counts of <10,000/μL. In such
cases, the leukemic cells often are reported initially to be atypical lymphocytes, and it is only on
further evaluation that the cells are found to be part of a malignant clone.
When the results of an analysis of peripheral blood suggest the possibility of leukemia, a
bone marrow examination should be done promptly to establish the diagnosis. Bone marrow
aspiration alone usually is sufficient, but sometimes a bone marrow biopsy is needed to provide
adequate tissue for study or to exclude other possible causes of bone marrow failure.
32
27. A child from primipregnancy was born in a term labor and has body weight of 4000 g and
body length of 57 cm. When born, he was nonresponsive to examination. Diagniosis is diffuse.
Heart rate is 80/min.What resuscitation measures should be taken?
A. Begin ALV with mask
B. Introduce 100%oxygen
C. Intubate and begin ALV
D. Tactile stimulation
E. Administer naloxone

Correct answer is A: Begin ALV with mask.


Indications for mechanical ventilation by resuscitation bag and mask:
 Absence / inadequate independent breathing, heart rate <100 for 1 minute
 Absence or inadequate independent breathing after the initial steps of care conducted
within 30 seconds after birth
 HR <100 per 1 minute regardless of the availability and adequacy of independent
breathing after the initial steps of care conducted within 30 seconds after birth.
 Persistent central cyanosis, despite the presence of adequate independent breathing,
heart rate> 100 for 1 minute and feed the free flow of 100% oxygen for at least 5
minutes.

28. A 15-year-old teenager has undergone medical examination in military recruitment center.
The following was revealed: interval systolic murmur at the cardiac apex, accent of the II heart
sound over the pulmonary artery, tachycardia. What additional examination method will be most
informative for determining a diagnosis?
A. Echocardiography
B. Electrocardiography
C. X-ray
D. Phonocardiography
E. Rheography

Correct answer is A : Echocardiography

The ACC, the AHA, and the American Society of Echocardiography (ASE) have published
detailed practice guidelines for the clinical application of echocardiography. More recently, these
and other bodies have collaborated to establish appropriate use criteria for echocardiography.
Briefly, indications of echocardiography may be divided into structural imaging and
hemodynamic imaging. Indications for structural imaging include the following:
 Structural imaging of the pericardium (eg, to exclude pericardial effusion)
 Structural imaging of the left or right ventricle and their cavities (eg, to evaluate ventricular
hypertrophy, dilatation, or wall motion abnormality; to visualize thrombi)
 Structural imaging of the valves (eg, mitral stenosis,aortic stenosis, mitral valve prolapse;
see the first image below)
 Structural imaging of the great vessels (eg, aortic dissection)
 Structural imaging of atria and septa between cardiac chambers (eg, congenital heart
disease, traumatic heart disease; see the second image below)

29. A 6-year-old girl attended a general practitioner with her mother. The child complains of
burning pain and itching in her external genitalia. The girl was taking antibiotics the day before

33
due to her suffering from acute bronchitis. On examination: external genitalia are swollen,
hyperemic, there is white deposit accumulated in the folds. The most likely diagnosis is:
A. Candidal vulvovaginitis
B. Trichomoniasis
C. Nonspecificvulvitis
D. Helminthic invasion
E. Herpes vulvitis

Correct answer is A - Candidal vulvovaginitis

Candidiasis is an infection caused by Candida. Candida normally is found on the skin,


vaginal area, and digestive system. But in some cases, it can overgrow. This can cause a rash,
itching, and other symptoms. Yeast normally lives on and in the body and causes no harm. It’s
found on the skin, in the digestive system (including the mouth and throat), and in the genital
area. But it can cause an infection in certain conditions. This can happen when the skin is
damaged, when it’s warm and humid, or when a child has a weak immune system. In some
children, it can infect deeper tissues or the bloodstream and cause serious illness. Medicine with
antibiotics or corticosteroids can also cause the yeast to overgrow.
Clinical manifestations of candidal vaginitis:
 White or yellow discharge from the vagina
 Itching
 Redness of the vulva
 Burning

Vulvovaginitis is generally considered to be the commonest gynaecological problem in


prepubertal girls. In practice, the terms vulvitis, vaginitis, and vulvovaginitis are often used
interchangeably by doctors in diagnosing inflammatory conditions of the lower female genital
tract. The hypo-oestrogenic hormonal milieu in a preadolescent girl is a major factor in making
her vaginal mucosa susceptible to infection.
Diagnosing infection is confounded by the overlap between normal flora and potential
pathogens. The presence of an organism does not itself denote causation, and the clinical picture
as well as microbiology should be considered before infection is assumed. Despite vulvovaginitis
being a common problem, the initial management is mostly empirical.
One of the most common causes of vulvovaginitis is Candida albicans. This  Candida
infection can cause genital itching and a thick, white vaginal discharge. A Candida infection
often follows the use of antibiotics. Antibiotics can kill the antifungal bacteria that normally live
in the vagina. This can lead to a yeast infection. Candida may be found in girls with predisposing
factors, such as a recent course of antibiotics.
Organisms associated with sexually transmitted diseases also can cause vulvovaginitis:
finding Neisseria gonorrhoeae or Chlamydia trachomatis  should prompt a careful evaluation for
sexual abuse. Viruses that can cause vulvovaginitis (include herpes and human papillomavirus)
are typically sexually transmitted.

30. A baby was born by a young smoker. The labour was complicated by uterine inertia,
difficult delivery of the baby’s head and shoulders. The baby’s Apgar score was 4. Which of the
following is a risk factor for a spinal cord injury?
A. Difficult delivery of the head and shoulders
B. Young age of the mother
C. Pernicious habits
D. Uterine inertia
E. Chronic hypoxia

34
Correct anser is A- Difficult delivery of the head and shoulders

 Explanation :
The delivery of a baby puts enormous pressure on a baby’s body, but typically this
pressure is not on the spinal cord.  Most pressure is applied laterally to the baby’s skull, and
because the baby’s skull is still soft and not fully formed, it is usually able to mold to the size of
the birth canal and suffer no serious injuries.
As the baby moves through the birth canal, spinal cord injuries may be caused by
hyperextension, improper rotation of the head, or traction and compression. Apgar score of 1-3
are critically low, 4-6 are below normal, and 7+ are normal. The shoulder getting stuck on the
mother’s pelvic bone (shoulder dystocia) all can increase the likelihood of head and spinal cord
trauma during delivery and can cause a low Apgar score.

31. A 15-year-old patient suffers from headache, nasal haemorrhages, sense of lower extremity
coldness. Objectively: muscles of shoulder girdle are developed, lower extremities are
hypotrophied. Pulsation on the pedal and femoral arteries is sharply dampened. BP is 150/90 mm
Hg, 90/60 in the legs. Systolic murmur can be auscultated above carotid arteries.What is the
most probable diagnosis?
A. Aorta coarctation
B. Aorta aneurism
C. Aortal stenosis
D. Aortal insufficiency
E. Coarctation of pulmonary artery

Correct anser is A – Aorta coarctation


Explanation :
Coarctation of the aorta, also called aortic narrowing, is a congenital condition whereby
the aorta is narrow, usually in the area where the ductus arteriosus (ligamentum arteriosum after
regression) inserts. Blood pressure is higher before the narrowing, and lower past the
narrowing.  Hypertension is defined when a patient's blood pressure in the arm exceeds 140/90
mmHg under normal conditions. Arterial hypertension in the arms with low blood pressure in the
lower extremities is classic symptom of Coarctation of the aorta . In the lower extremities weak
pulses in the femoral arteries and arteries of the feet are found.  Radio-femoral delay will be
present under palpation in either arm (both arm pulses are normal compared to the delayed leg
pulses).

32. A 15-year-old boy has developed pain in the area of his left knee joint. Objectively: thigh
soft tissues in the painful area are infiltrated, joint function is reduced. X-ray: there is a focus of
destruction in the left distal femoral metaphysis, with periosteum detachment and formation of
Codman triangle in the bone cortical layer at the margin of the defect. Chest X-ray reveals
numerous microfocal metastases. What pathology is the most likely to cause such presentation?
A. Osteogenic sarcoma
B. Fibrosarcoma
C. Chondrosarcoma
D. Ewing’s sarcoma
E. Juxtacortical sarcoma

Correct anser is A: Osteogenic sarcoma

Explanation :

35
Osteosarcoma is the third most common cancer in adolescence, occurring less frequently
than only lymphomas and brain tumors. It is thought to arise from a primitive mesenchymal
bone-forming cell and is characterized by production of osteoid.
Osteosarcoma is a bone tumor that can occur in any bone. It most commonly occurs in
the long bones of the extremities near metaphyseal growth plates. The most common sites
include the femur (42%), with 75% of tumors in the distal femur. Any sarcoma that arises from
bone is technically called an osteogenic sarcoma. Therefore, this term includes fibrosarcoma,
chondrosarcoma, and osteosarcoma, all named for their morphologic characteristics. Ewing’s
family of tumors most frequently occurs in teenagers. It can occur in any bone in the body, but it
is most common in the pelvis, thigh, lower leg, upper arm, and rib. Metastasis is when cancer
spreads from where it started to another part of the body. Whether or not the cancer has already
spread when it is first diagnosed is the most important prognostic factor for childhood
osteosarcoma. Metastasis is linked with a poorer prognosis. Children with osteosarcoma that has
only spread to a lung have a more favourable prognosis than those with metastasis in other parts
of the body.

33. Head circumference of a 1-month-old boy with signs of excitement is 37 cm, prefontanel is
2x2 cm large. After feeding the child regurgitates small portions of milk; stool is normal in its
volume and composition. Muscle tone is within norm. What is the most likely diagnosis?
A. Pylorospasm
B. Meningitis
C. Pylorostenosis
D. Microcephaly
E. Craniostenosis

Correct anser is A: Pylorospasm

Explanation :
1-month-old boy has normal head circumference, an increased size of the anterior
fontanel and minimal neurological symptoms. Pylorospasm is condition characterized by gastric
spasms opening into the 12-colon. With this disease, passage of food through the gastrointestinal
tract is difficult, which complicates the outflow of the contents to the outside. This reflex can
occur immediately after feeding or after 10-15 minutes. In this case, the secretions resemble
cheesy milk. Their mass exceeds the amount of food consumed by the child. Sometimes
vomiting does not occur, but uneven regurgitation is observed. The child slowly gaining weight.
Possible disturbance of stool in a newborn. With frequent regurgitation, the volume of urine
decreases. The general condition of the child is satisfactory. The reasons for the development of
pylorospasm include: disrupted secretion of digestive glands, insufficient quantity of hormones
of the digestive tract, hathology of the CNS.
The symptomatology of this disorder is similar to stenosis of the stomach. Pain and
discomfort make the child moody and whiny. In addition, there is a constant craving for
vomiting. With the progression of the disorder, the baby loses weight, weakens. Regurgitation
becomes permanent. The height of the disease is characterized by frequent vomiting, dry skin,
dehydration. This distinguishes gastric stenosis from ordinary pylorospasm.

34. 10 days after birth an infant developed a sudden fever up to 38, 1oC. Objectively: the skin of
navel, abdomen and chest is erythematous; there are multiple pea-sized blisters with no
infiltration at the base; single bright red moist erosions with epidermal fragments on the
periphery.What is your provisional diagnosis?
A. Epidemic pemphigus of newborn
B. Syphilitic pemphigus
36
C. Streptococcal impetigo
D. Vulgar impetigo
E. Atopic dermatitis

Epidemic pemphigus newborns- highly contagious staphylococcal skin lesion newborns,


manifested appearance of fine bubbles with purulent contents, which increase and bursting with
the formation of erosions. It may be accompanied by lesions of the mucous membranes.
Diagnose epidemic pemphigus newborns allows age of onset, the typical clinical picture and
results of bacteriological studies of fluid from blisters and erosions discharge.

35. A 15-year-old patient complains of excessive body weight, headache, irritability, rapid
fatigability. Significant increase of body weight occurred at the age of 14. Objectively: weight is
90 kg; height is 160 sm, proportional body built. Fatty tissue is distributed evenly. There are thin
pink striae (stretch marks) on the thighs, abdomen and mammary glands. BP - 145/90 mm Hg.
Provisional diagnosis is:
A. Pubertate dyspituitarism
B. Alimentary constitutive obesity
C. Somatoform autonomic dysfunction
D. Itsenko-Cushing’s disease
E. Cushing’s syndrome

Correct anser is A: Pubertate dyspituitarism


Explanation :
Dyspituitarism - abnormal condition caused by dysfunction of the pituitary gland.
Childhood overweight (body mass index (BMI)>90th centile) poses a major public health
problem in so far as adult diseases manifest themselves already during childhood.  Applying
WHO criteria, obese children showed significantly higher levels of insulin, homeostasis model
assessment index and most vascular parameters, as well as lower high-density lipoprotein
(HDL)-cholesterol than overweight children. Moreover, overweight children showed higher BP,
insulin and uric acid, and lower HDL-cholesterol than normal weight children.
Childhood obesity has contributed to an increased incidence of type 2 diabetes mellitus
and metabolic syndrome (MS) among children. Higher blood pressures and a higher prevalence
of hypertension occur in overweight children, independent of race, sex, and age.
Pseudo- Cushings syndrome (PCS) is a group of conditions associated with clinical and
biochemical features of Cushing syndrome, but the hypercortisolemia is usually secondary to
other factors. A particular type of hypercortisolism presenting during the different conditions
(obesity, eating disorders and other), named functional hypercortisolism, is caused by chronic
activation of hypothalamic-pituitary-adrenal (HPA) axis. It is usually mild and disappears when
the underlying disorder is resolved.
The metabolic syndrome (hypertension, glucose intolerance, hypertriglyceridemia,
decreased high-density lipoprotein level, abdominal central obesity) confers an excessively high
risk of cardiovascular disease, with an overall prevalence of 4% in adolescents and
approximately 30% in overweight adolescents. Hypothyroidism can be associated with obesity,
but usually weight gain is modest, because appetite is often reduced and problems of poor linear
growth, delayed skeletal development, and delayed puberty are more prominent features.
The onset of relatively rapid weight gain, an increase in BMI percentiles, and central
obesity in a child or adolescent may occur with Cushing syndrome, but other symptoms or
findings, such as muscle weakness, ecchymoses, unexplained osteoporosis, and hypokalemia
may be present. Cushing's syndrome refers to excess cortisol of any etiology
(as syndrome means a group of symptoms). One of the causes of Cushing's syndrome is a
cortisol-secreting adenoma in the cortex of the adrenal gland (primary
hypercortisolism/hypercorticism). Cushing's disease refers only to hypercortisolism secondary to
37
excess production of ACTH from a corticotroph pituitary adenoma (secondary
hypercortisolism/hypercorticism) or due to excess production of hypothalamus CRH
(Corticotropin releasing hormone) (tertiary hypercortisolism/hypercorticism).

36. When playing in a kindergarten a 3-year-old child sudenly developed dyspnea, paroxysmal
compulsive dry cough. The face is cyanotic, the eyes are tearful. Vomiting occurred several
times. Breathing is weakened over the whole right side of the chest. The provisional diagnosis is:
A. Foreign body
B. Obstructive bronchitis
C. Bronchial asthma
D. Hysteria fit
E. Stenosing laryngotracheitis

Correct anser is A: Forein body


Explanation :
Foreign body airway obstruction ia a partial or complete blockage of the breathing tubes
to the lungs due to a foreign body (for example, food, a bead, toy, etc.). The onset of respiratory
distress may be sudden with cough. There is often agitation in the early stage of airway
obstruction. The signs of respiratory distress include labored, ineffective breathing until the
person is not longer breathing (apneic). Loss of consciousness occurs if the obstruction is not
relieved. A foreign body may become lodged in the larynx, trachea, or bronchus.
The right bronchus is more commonly affected than the left because of the lesser angle of
divergence relative to the left bronchus and because of its greater diameter. Affected patients
may experience little acute distress after an initial phase of choking episode. A history of
choking in a previously healthy child can be elicited in 80% to 90% of patients who have
aspirated a foreign body. Physical examination results may be normal or nonspecific  in up to
30% patients. Others may have generalized or localized wheezing and decreased air entry.
Because foreign-body aspiration can mimic other respiratory conditions, a high index of
suspicion is  necessary in all patients with pneumonia, atelectasis, or wheezing with an atypical
course—especially in patients who are unresponsive to medical therapy. Foreign-body
aspiration can closely mimic an acute asthma exacerbation. However, the absence of atopy, the
acute onset of symptoms, and unilateral physical findings may suggest foreign-body aspiration.

37. For the last 3 years a 12-year-old boy has been suffering from stomachache, abdominal
distension, nausea, periodical liquid fatty stool, grey in color, with rotten smell. On palpation:
pain in the epigastrium, Desjardins’ pancreatic point and Chauffard’s triangle; positive
MayoRobson’s sign. Insufficiency of pancreas exocri-ne function is suspected. What method is
the most informative for pancreas exocrine function assessment?
A. Detection of elastase-1 in feces
B. Determining tripsin content in blood serum
C. Pancreas echography
D. Determining amylase content in blood and urine
E. Scatological test

Correct anser is A: Detection of elastase-1 in feces.


Explanation :
Exocrine pancreatic insufficiency (EPI) is characterized by a deficiency of exocrine
pancreatic enzymes, resulting in malabsorption. Numerous conditions account for the etiology of
EPI, with the most common being diseases of the pancreatic parenchyma including chronic
pancreatitis, cystic fibrosis, and a history of extensive necrotizing acute pancreatitis. Conditions

38
that cause pancreatic insufficiency in children include cystic fibrosis, Shwachman-Diamond
syndrome, isolated enzyme deficiencies, chronic pancreatitis, and protein-calorie malnutrition.
For most patients with chronic pancreatitis, abdominal pain is the presenting symptom.
The patient experiences intermittent attacks of severe pain, often in the mid-abdomen or left
upper abdomen and occasionally radiating in a bandlike fashion or localized to the midback. The
pain may occur either after meals or independently of meals, but it is not fleeting or transient and
tends to last at least several hours. Other symptoms associated with chronic pancreatitis include
diarrhea and weight loss.
Pancreatic functional status has a very strong effect on outcome in pancreatic
insufficiency requires lifelong treatment with pancreatic enzymes. Fecal elastase has good
sensitivity, specificity, and positive and negative predictive value for defining severe pancreatic
insufficiency in patients and appears to be more useful than measurement of other fecal enzymes.

38. A 15-year-old girl complains of dizziness and sensation of lack of air that she develops in
emotionally straining situations. Relief occurs after she takes corvalol.Objectively: hyperhidrosis
and marble-like pattern of the skin of her palms and feet. Clinical and instrumental examination
revealed no organic alterations of the central nervous, cardiovascular, and respiratory
systems.What provisional diagnosis can be made?
A. Somatoform autonomic dysfunction
B. Obstructive bronchitis
C. Bronchial asthma
D. Stenosing laryngotracheitis
E. Acute epiglottitis

Correct anser is A: Somatoform autonomic dysfunction


Explanation :
The somatoform disorders are a group of psychological disorders in which a patient experiences
physical symptoms that are inconsistent with or cannot be fully explained by any underlying
general medical or neurologic condition. Medically unexplained physical symptoms account for
as many as 50% of new medical outpatient visits. Physical symptoms or painful complaints of
unknown etiology are fairly common in pediatric populations.   Many healthy young children
express emotional distress in terms of physical pain, such as stomachaches or headaches, but
these complaints are usually transient and do not effect the child's overall functioning. The
somatoform disorders represent the severe end of a continuum of somatic symptoms. The
essential feature of a somatization disorder is a pattern of many physical complaints in persons
younger than 30 years that occurs over several years and results in unnecessary medical
treatment and/or causes significant impairment in functioning. This diagnosis was historically
referred to as hysteria or Briquet syndrome. [9] The somatic symptoms are neither intentionally
produced nor feigned and appear to be unconscious to the patient. All the following historical
criteria are required for a diagnosis [7] :
 Four different pain sites (eg, head, abdomen, back, joints, extremities, chest, rectum) or
painful functions (eg, menstruation, sexual intercourse, urination)
 Two gastrointestinal symptoms other than pain (eg, nausea, bloating, vomiting, or
intolerance of several different foods)
 One sexual or reproductive symptom other than pain (eg, erectile or ejaculatory
dysfunction, irregular menses, excessive menstrual bleeding)
 One pseudoneurological symptom (eg, impaired balance, paralysis, aphonia, urinary
retention)
The criteria for somatization disorder were designed for adults, and attempts have been made to
apply criteria to pediatric populations since adolescents can present to primary care facilities
with many of these symptoms. Nevertheless, this diagnosis is rarely made in the adolescent

39
population, mainly because of the time requirement of several years that is needed to meet the
symptom criteria.

39. A 14-year-old patient complains of alopecia foci on his scalp. The patient has
been presenting with this condition for 2 weeks. Objectively: on the scalp there are
several small oval foci with blurred margins. The skin in the foci is pink-red, the hairs are
broken off at 4-5 mm length or at skin level. UnderWood’s lamp there are no foci of green
luminescence detected.What disease is it?
A. Trichophytosis capitis
B. Syphilitic alopecia
C. Alopecia areata
D. Scleroderma
E. Psoriasis

Correct anser is A: Trichophytosis capitis.


Explanation :
Tinea capitis is the most common type of superficial dermatophytosis in children, affecting the
skin and hair of the scalp. The disease occurs most often in prepubertal children and is
characterized by scaling and hair loss of the scalp. The most common dermatophytes that cause
tinea capitis include Trichophyton tonsurans (most prevalent in the United States), T. violaceum,
Microsporum canis and M. audouinii (most prevalent in the world outside the United States).
The most common clinical findings include the following:
1. Alopecia +/- scale (localized or generalized)
2. Posterior cervical and/or posterior auricular lymphadenopathy
3. Hairs broken near the scalp ("black dots") - only with T. tonsurans infection
4. Diffuse scale - This can mimic seborrheic dermatitis, with widespread scale throughout the scalp.
Seen more commonly in African Americans.
There are three type of tinea capitis, microsporosis, trichophytosis, and favus; these are based on
the causative microorganism, and the nature of the symptoms. In microsporosis, the lesion is a
small red papule around a hair shaft that later becomes scaly; eventually the hairs break off 1–
3 mm above the scalp. This disease used to be caused primarily by Microsporum audouinii, but
in Europe, M. canis is more frequently the causative fungus. The source of this fungus is
typically sick cats and kittens; it may be spread through person to person contact, or by sharing
contaminated brushes and combs. In the United States, Trichophytosis is usually caused by
Trichophyton tonsurans, while T. violaceum is more common in Eastern Europe, Africa, and
India. This fungus causes dry, non-inflammatory patches that tend to be angular in shape. When
the hairs break off at the opening of the follicle, black dots remain. Favus is caused by T.
schoenleinii, and is endemic in South Africa and the Middle East. It is characterized by a number
of yellowish, circular, cup-shaped crusts (scutula) grouped in patches like a piece of honeycomb,
each about the size of a split pea, with a hair projecting in the center. These increase in size and
become crusted over, so that the characteristic lesion can only be seen around the edge of the
scab.

40. A 12-year-old girl after a case of respiratory infection developed dyspnea at rest,
paleness of skin. Heart rate is 110/min., BP is 90/55 mm Hg. Heart sounds are muffled.
Borders of relative heart dullness: right – the parasternal line, upper - the III rib, left -
1,0 cm outwards from the midclavicular line. Make the provisional diagnosis:
A. Infectious myocarditis
B. Functional cardiopathy
C. Somatoform autonomic dysfunction
D. Hypertrophic cardiomyopathy
E. Exudative pericarditis
40
Correct anser is A: Infectious myocarditis
Explanation :
Myocarditis is rare in young children. It is slightly more common in older children and adults. It
is often worse in newborns and young infants than in children over age 2. Most cases in children
are caused by a virus that reaches the heart. These can include the influenza (flu) virus,
Coxsackie virus, parovirus, and adenovirus. It may also be caused by bacterial infections such
as Lyme disease.
Other causes of pediatric myocarditis include:
 Allergic reactions to certain medicines
 Exposure to chemicals in the environment
 Infections due to fungus or parasites
 Radiation
 Some diseases (autoimmune disorders) that cause inflammation throughout the body
 Some drugs
Dyspnea with feeding, vomiting and pallor are common presenting complaints in infants. Older
children present with a marked decrease in stamina and shortness of breath in the context of a
viral infection. Complaints of palpitation suggest coexistent arrhythmia. Chest pain is
nonspecific and indicates a possible associated pleural or pericardial inflammation. On physical
examination, the child is tachypneic, tachycardic and febrile. Hypotension suggests severely
depressed cardiac function. Pallor and peripheral vasoconstriction due to sympathetic activation
are usually noted. Peripheral capillary refill is delayed. In children, the earliest sign of fluid
retention secondary to congestive heart failure is eyelid puffiness. More severe failure presents
with hepatomegaly and generalized edema. A pericardial rub and distant heart sounds suggest
the presence of an associated pericardial effusion. A soft first heart sound is common but
nonspecific and often difficult to appreciate in the tachycardic infant. A gallop rhythm produced
by a third heart sound is frequent and best heard at the apex. A high pitch regurgitant murmur
detected at the apex indicates coexistent mitral insufficiency. The 12-lead electrocardiogram
typically shows sinus tachycardia and low voltage QRS complexes (less than 5 mm total
amplitude in all limb leads), with lowamplitude or slightly inverted T waves, and a small or
absent Q wave in leads V5 and V6. PR and QT interval prolongation are common but
nonspecific. The presence of arrhythmia in the context of a febrile illness should point the
clinician to the possibility of myocarditis. Paroxysmal atrial tachycardia ventricular ectopy and
various degrees of heart block have all been described in association with myocarditis. The
echocardiogram is essential in the diagnosis and follow-up of myocarditis. Structural heart
disease and pericarditis can be readily ruled out. Myocardial function is assessed at the time of
diagnosis, and serves as a baseline for future monitoring of disease progression and response to
therapeutic interventions. Isolation of the offending virus from stool, throat washings or blood is
possible early in the disease . Otherwise, serological diagnosis relies on a fourfold increase in
virus-specific antibody titres between acute and convalescent blood samples. The role of
myocardial biopsy in establishing the diagnosis of myocarditis is controversial. Endomyocardial
biopsy can establish the diagnosis and degree of involvement. Polymerase chain reaction
performed on myocardial tissue has been recently shown to be both sensitive and specific to the
diagnosis of enteroviral and adenoviral myocarditis . The utility of biopsy must be weighed
against the risk of perforation and death, particularly in the ill infant. In addition, patchy right
ventricular disease or isolated left ventricular involvement can escape detection when random
biopsy samples are limited to the right ventricular septum.

41

You might also like